Sie sind auf Seite 1von 168





PREFACE

Welcome to UDAAN - a program to give wings to girl students!

The UDAAN programme has been initiated with the primary objective of increasing
the enrollment of girl students in leading engineering institutions. The program is
designed to provide a comprehensive platform to deserving girls who aspire to pursue
higher education in engineering and assist them in preparing for the entrance
examinations and increasing their probability of being a student in the most prestigious
engineering institutions of the country.

This study material provided to you is aligned with the tutorials given to you on your
tablets and made available online. A soft copy of this material is available on your
tablets as well. You will find that the material has been simplified and made easy to
read. Each topic has been divided into Subtopics to relate it with the tutorials. Each
subtopic discussed is followed by some practice questions for you to attempt. Answers
to these questions are given at the end. In case you have difficulty in solving any of
them, please contact your helpdesk and seek clarifications.

You can call 1800 180 7542 or write to udaan.cbse@gmail.com for any queries.

Best of luck and happy learning !

Chairperson, CBSE
Contents
S.No. Topics Page Number

1. Coordinate Geometry - I 1
2 Coordinate Geometry - II 5
Practice Questions 9
3. Coordinate Geometry - III 11
4. Coordinate Geometry - IV 19
Practice Question 20
5. Coordinate Geometry - V 23
Practice Question 25
6. Coordinate Geometry - VI 27
7. Coordinate Geometry - VII 33
8. Coordinate Geometry - VIII 37
Practice Question 41
9. Circle: Equation of Circles - I 45
10. Circle: Position of a Point - II 49
11. Circle: Problem Solving - III 51
Practice Question 54
12. Circle: Tangents and Normals - IV 57
13. Circle: Tangents and Normals - V 61
14. Circle: Tangents and Normals - VI 65
Practice Questions 75
15. Circle: Intersection of Two Circles - VII 79
16. Circle: Radical Axis - VIII 85
17. Circle: -Co- Axial System of Circles - IX 91
18. Circle: Limiting Points- X 95
19. Circle: Problem Solving - IX 99
Practice Questions 106
20. Parabola Conic Section - I 111
Practice Questions 114
Contents
S.No. Topics Page Number
21. Parabola Parabola - II 117
Practice Questions 120
22. Parabola -III 121
Practice Questions 123
23. Parabola Parabola - IV 125
Practice Questions 128
24. Parabola Parabola - V 129
25. Parabola Parabola - VI 135
Practice Questions 139
26. Parabola Parabola - VII 141
27. Parabola Parabola - VIII 143
28. Parabola Parabola - IX 147
Practice Questions 150
29. Parabola Parabola - X 153
Practice Questions 157
COORDINATE GEOMETRY-I
Straight Line
Topics covered
1. Slope of a straight line
2. Angle between two lines
3. Special forms of straight lines
(i) Slope and point form
(ii)Two point form
(iii) Slope and y-intercept form
Straight Line -
It is a curve such that every point on the line segment joining any two points on it lies on it.
Equation of straight line is always of first degree on x and y.
General equation of straight line is ax+by+c=0
Where a,b,c∈R.a and b can be zero but not both at the same time .
Slope of a line
The trigonometrical tangent of an angle that a line makes with the positive direction of the x-
axis in anticlockwise direction is called the slope or gradient of the line
Normally slope is denoted by m= tan θ .
y y

θ θ
O x O x

Points to remember
(i) Slope of a line parallel to x-axis is zero and perpendicular to x-a xis is undefined.
(ii) If slope is positive then it makes acute angle with positive direction of x-axis and nega-
tive it will make obtuse angle with positive direction of x-axis.
(iii) If a line is equally inclined to the axes, that it will make angle of 450 or 1350, with x-
axis. Hence slope of line is + 1.
(iv) It three points A,B,C are collinear then slope of AB=slope of BC=slope of AC
y1 − y2 y2 − y1
(v) Slope of the line joining two points (x1, y1) and (x2, y2) is m= x − x or x − x
1 2 2 1

y B (x , y )
2 2

(x1, y1)
A N
θ
O C M x

(vi) Equation of x-axis is y=0


(vii) Equation of y-axis is x=0

1
y
y=a (a>0)

x
O

y=a (a<0)
(viii) Equation of line parallel to x-axis is y = a, where a is any real number
(ix) Equation of line parallel to y-axis (perpendicular to x-axis) is x=a, where a is any real
number
y

O x

x=a(a<0) x=a(a>0)
Angle between two lines
The angle θ between the lines having slopes m1 and m2 is given by
m1 − m 2 ⎛ m − m2 ⎞
tan θ = = +⎜⎜ 1 ⎟⎟
1 + m1m 2 ⎝ 1 + m1m 2 ⎠
y
B A
P
θ θ2
θ1
O x1

Where m1=tan θ 1, m2 =tan θ 2


(i) When two lines are parallel then their slopes are equal ie. m1= m2
(ii) When two lines are perpendicnlar then the product of their slope is -1
i.e. m1m2=-1 or 1+m1m2=0
A line equally inclined with two lines
Let the two lines with slopes m1 and m2 be equally inclined to a line with slope m then
⎛ m1 − m ⎞ ⎛ m −m ⎞
⎜⎜ ⎟⎟ = −⎜⎜ 2 ⎟⎟
⎝ 1 + m1 m ⎠ ⎝1+ m2m ⎠
m1
m
θ
θ
m1
Special form of a line
1. Point slope form
The equation of a line which passes through the point (x1,y1) and has the slope m is y-y1=m(x-
x1)

2
y P(x, y)

A(x1, y1)

o x

2. Two point form


The equation of a line passing through two points (x1,y1) and (x2,y2) is
y 2 − y1
y-y1= (x − x 1 )
x 2 − x1
y P(x, y)
B(x2, y2)
A(x1 , y1)

o x

3. Slope and y intercept form [Non Vertical Line]


The equation of a line with slope m that makes an intercept c on y-axis is y=mx+c
y

(o, c)
θ
O x

Example

1. Find the equation of a line that has y-intercept -3 and is perpendicular to the line joining (2,-
3) and (4,2).
Solution
y 2 − y1 2 + 3 5
Slope of line joining the points (2,–3) and (4,2) is x − x = 4 − 2 = 2
2 1

Since the required line is perpendicular to it

( )
∴ (slope of the required line) X 5 2 = -1

∴ slope of the required line = − 2 5 =m


y-intercept = -3
Equation of line is y = − 2 5 x − 3
2x+5y+15 = 0

3
Examples:
1. If the centroid and circumcentre of a triangle are (3,3) and (6,2) respectively, then the orthocentre
is
(a) (-3,5) (b) (-3,1) (c) (3,-1) (d) (9,5)
Solution: Centroid, circumcentre and orthocenter are collinear such that centroid divides the
circumcentre and orthocentre in the ratio 1:2. Ans(-3,5).
2. If the algebraic sum of the perpendicular distances from the point (2,0), (0,2) and (1,1) to a
variable straight line be zero, then the line passes through the point
(a) (3,3) (b) (1,1) (c) (1,-1) (d) (-1,-1)
Solution: Let equation be ax+by+c=0
2a + ob + c 0 + 2b + c a+b+c
A.T.Q. + + =0
a +b
2 2
a +b 2 2
a 2 + b2
⇒3a+3b+3c=0 ⇒ a+b+c=0
This shows that ax+by+c=0 passes through (1,1).
3. The area enclosed by 2|x|+3|y|≤6 is
(a) 3 Sq-units (b) 12 Sq.units (c) 9 Sq.units (d) 24Sq.units
Solution: 2x+3y ≤ 6, x≥0, y≥0
2x-3y≤ 6, x≥0, y≤0
-2x+3y ≤ 6, x≤0, y≥0
-2x-3y ≤ 6, x≤0, y≤0
Form a rhombus with diagonals 4 & 6.
1
Area = ×4×6= 12 sq. units.
2
3

4
COORDINATE GEOMETRY-II
Straight Line
Topics Covered
1 Intercept form
2 Normal form
3 Distance form OR Symmetric form OR Parametric form
Intercept form
The equation of a straight line which cuts off intercepts a and b on x–axis and y–axis respec-
tively is given by
y y y y
(0, b)
(0, b)

x x
(–a, 0) O O (a, 0)
x x
O (a, 0) O
(–a, 0) (0, –b) (0, –b)

x y –x y x y x y
+ =1 + =1 + = –1 – =1
a b a b a b a b
Normal form (perpendicular form)
The equation of a straight line upon which the length of the perpendicular from the origin is p and
the perpendicular makes an angle α with the positive direction of x–axis is given by
xcos α +ysin α = p
y

p
α
x
O A

Note Here p is always taken as positive and α is measured from positive direction of x–axis in
anticlockwise direction between 0 and 2 π
Distance form or Symmetric form or Parametric Form
The equation of a straight line passing through the point (x1, y1 ) and making an angle θ with the
positive direction of x–axis is given by y
x – x 1 y – y1
= =r r P (x,y)
cos θ sin θ θ
B (xA1,y1)
Where r is the distance of the point (x, y) from the point (x1,y1)
Parametric form θ
x
From the above equation we get O
x–x1 = rcos θ , y–y1 = rsin θ
x = x1+rcos θ y=y1+rsin θ
5
The coordinates (x,y) of any point on the line at a distance r from the point A (x1,y1) can be taken as
(x1+rcos θ , y1+rsin θ )
Where the line is inclined at an angle θ with positive direction of x–axis.

1 If P is on the right side of A(x1,y1) then r is positive and if p is on the left side of A(x1,y1) then
r is negative.
x – x1 y – y1
2 At a given distance r from the point (x1,y1) on the line = , there are two points
cos θ sin θ
viz; (x1+rcos θ ,y1+rsin θ ) and (x1–rcos θ ,y1–rsin θ )
Examples
x y
1 Through the point P( α 1 β ) where α β >0 the straight line + = 1 is drawn so as to form with
a b
coordinate axes a triangle of area S. If ab>0 then the least value of S is
(a) α β (b) 2 α β (c) 4 α β (d) None.
Solution
x y
Given equation of line is + = 1 ____________________(1)
a b
area of Δ OAB = S
1
ab = S
2
Or ab = 2S ab>0
Equation (1) passes through the point P( α , β )
α β α aβ
+ =1 or + =1
a b a ab
α aβ
y + =1
a 2S
B(0, b)
a2 β –2aS+2S α = 0
P(α,β)
b
S a is real
A(a, 0) x
O a ∴D≥0
4S2–8S α β ≥ 0
S2–2S α β ≥ 0
S≥2α β
∴ least value of S is 2 α β .
2 The distance of the point (2,3) from the line 3x+2y =17 measured parallel to the line x-y=4 is
(a) 4 2 (b) 5 2 (c) 2 (d) None.
Solution.
Any point of the line parallel to x–y = 4 and passes through P(2,3) at a distance r, are

6
π
(2+rcos θ ,3+rsin θ ) where tan θ = 1=slope of line x–y =4 therefore θ = 45° =
4
This point lies on the line 3x+2y = 17
∴ 6+3r cos45° + 6 + 2r sin45° = 17
P(2, 3)
3r 2r x–y=4
6+ +6+ = 17
2 2 3x+2y=17

5r
=5
2
r
=1
2
r= 2
3 A ray of light travelling along the line x+ 3 y = 5 is incident on the x-axis and after refraction it
enters the other side of the x–axis by turning π /6 away from the x–axis. The equation of the line
along which the refracted ray travels is
(a) x+ 3 y–5 3 = 0 (b) x– 3 y–5 3 = 0
(c) 3 x+y–5 3 =0 (d) 3 x–y–5 3 =0
Solution : The refracted ray passes through the point (5, 0) and makes an angle 1200 with positive
direction of x–axis Retracted Ray
y

∴ The equation of the refracted ray is


y–0 = tan1200(x–5) b

y = – 3 (x–5) (5. 0)

.16

3 x+y–5 3 = 0 Retracted Ray

4 Column Matching (This Q. is not relevant as option the righthand coloumn are unique)
easily identifide.
Column I Column II
1 It a, b, c are in H.P, then the straight line
x y 1 π
+ + = 0 always passes through a (a)
a b c 2
fixed point
2 The larger of the two angles made with
the axis of a straight line drawn through (b) (1, –2)
(1, 2) so that it intersects x+y = 4 at a
6
point distant from (1, 2) is
3
3 The diagonals of the parallelogram
whose sides are l x+my+n = 0,
lx+my+n1 = 0, mx+ly+n = 0 (c) Rhombus

7
mx+ly+n1 = 0 include an angle
4 The diagonals of a parallelogram
PQRS are along the lines x+3y = 4and
6x–2y = 7. Then PQRS must be a (d) 5π
12
Solution :
1 1 1
1 a, b, c are in H.P so , , are in A.P
a b c
2 1 1
∴ = +
b a c
1 2 1 x y 1
or – + =0= + +
a b c a b c
on comparing x = 1, y = –2 so coordinate (1,–2)
y
6
2 AP = r =
3 r
P
θ
x –1 y –2 A(1,2)

Equation of AP ⇒ = =r x
cos θ sin θ O x + y=4

∴ P (1+r cos θ , 2+rsin θ )


P will satisfy x+y = 4
6 6
∴ 1+ cos θ + 2+ sin θ = 4
3 3
π 3
cos( θ – )= = cos30°
4 2
5π π
∴ θ = 75° or 15° ie, or
12 12
3 Since the distance between parallel lines Lx+my+n=0 and Lx+my+n’=0 is same as the
n – n′
distance between the parallel lines mx+ly+n = 0 and mx+ly+n1 = 0. ie, there
L2 + m 2
fore the parallelogram is a rhombus. Since the diagonals of a rhombus are at right angles,
π
therefore the required angle is . =4
2 3y
S x+
R
–1
4. Slope of line x+3y = 4 is
3
6
slope of line 6x–2y = 7 is = 3
–1 2
6x

P P
–2
y=

product of slopes = × 3 = –1
7

3
8
∴ diagonals are perpendicular to each other
Answers
1 ⇔b 2 ↔d
3 ↔a 4 ⇔c
PRACTICE QUESTIONS
1 Orthocenter of triangle with vertices (0, 0), (3, 4) and (4, 0 ) is
⎛ 5⎞ ⎛ 3⎞
(a) ⎜ 3, ⎟ (b) (3, 12) (c) ⎜ 3, ⎟ (d) (3, 9)
⎝ 4⎠ ⎝ 4⎠
2 Area of the triangle formed by the line x+y = 3 and angle bisectors of the pairs of straight lines
x2–y2 + 2y = 1 is
(a) 2 sq.units (b) 4 sq.units (c) 6sq.units (d) 8 sq.units
3 Let O (0, 0), P(3, 4), Q (6, 0) be the vertices of the triangle OPQ. The point R inside the Δ OPQ
is such that the triangle OPR, PQR, OQR are of equal area. The coordinates of R are
⎛4 ⎞ ⎛ 2⎞ ⎛ 4⎞ ⎛4 2⎞
(a) ⎜ ,3 ⎟ (b) ⎜ 3, ⎟ (c) ⎜ 3, ⎟ (d) ⎜ , ⎟
⎝3 ⎠ ⎝ 3⎠ ⎝ 3⎠ ⎝3 3⎠
4 Consider three points P(–sin( β – α ),–cos β ), Q(cos( β – α ), sin β ) and R(cos( β – α + θ ), sin( β –
π
θ )), where 0< α , β , θ < 4 . Then
(a) P lies on the line segment RQ.
(b) Q lies on the line segment PR.
(c) R lies on the line segment QP.
(d) P, Q, R are non-collinear.
5 The locus of the orthocenter of the triangle formed by the lines (1+p) x– py + p (1+p) = 0, (1+q)x
– qy + (1+q)q = 0 and y = 0, where p ≠ q, is
(a) a hyperbola (b) a parabola (c) an ellipse (d) a straight line.
6 Let points A (1, 1) and B (2, 3). Coordinates of the point P such that |PA–PB| is minimum are
⎛ 3⎞ ⎛ 11 ⎞ ⎛3 ⎞
(a) ⎜ 2, ⎟ (b) ⎜ 0, ⎟ (c) (11, 3) (d) ⎜ ,0 ⎟
⎝ 2⎠ ⎝ 4⎠ ⎝2 ⎠
π
7 The line x+7y = 14 is rotated through an angle in the anticlock wise direction about the point (0,
4
2). The equation of the line in its new position is
(a) 3x–4y+8 = 0 (b) 3x–4y–8 = 0 (c) 4x+3y+8 = 0 (d) None of these
x y
8 If one diagonal of a square is the portion of the line + = 1 intercepted by the axes, then one
a b
to the extremities of the other diagonal of the square are
⎛a +b a – b⎞ ⎛a – b a +b⎞ ⎛a –b b–a⎞ ⎛a +b b–a⎞
(A) ⎜ , ⎟ (b) ⎜ , ⎟ (c) ⎜ , ⎟ (d) ⎜ , ⎟
⎝ 2 2 ⎠ ⎝ 2 2 ⎠ ⎝ 2 2 ⎠ ⎝ 2 2 ⎠

9
9 The image of P(a, b) in the line y = –x is Q and the image of Q in the line y=x is R, then the midpoint
of PR is
⎛a +b b+a⎞
(a) (a–b, b+a) (b) ⎜ , ⎟ (c) (a–b, b–a) (d) (0, 0)
⎝ 2 2 ⎠
10 Let ABC be a triangle. Let A be the point (1, 2), y = x is the perpendicular bisector of AB and x–
2y+1 = 0 is the angle bisector of ∠ C. If equation of BC is given by ax+by – 5 = 0, then a+b is
(a) 1 (b) 2 (c) 3 (d) 4

ANSWERS
1 c 2 a 3 c 4 d 5 d 6 b
7 a 8 c 9 d 10 b

10
COORDINATE GEOMETRY-III
Straight Line
Topics Covered
1 Distance of a line from a point
2 Distance between two parallel lines
3 Area of a parallelogram
4 Area of triangle
1 Distance of a line from a point
Length of perpendicular (distance) from the point (x1, y1) to the line ax+by+c = 0 is given by
| ax1 + by1 + c |
p =
a 2 + b2
y
c
0,–
B b
P(x1,y1)

H c
– ,0
A a
x
O

Length of perpendicular from the origin to the line ax+by+c = 0 is given by


|c|
p= , ( x1, y1) is (0, 0)
a 2 + b2
y
c
0 ,–
b

μ
b p
c
– ,0
a

a x
O

2 Distance between two parallel lines


The distance between two parallel lines ax+by+c1 = 0 and ax+by+c2 = 0 is given by
ax+by+c1=0

| c1 – c 2 |
p=
a 2 + b2
ax+by+c2=0

11
Note that coefficient of x and y of both equation must be same.
3 Area of a parallelogram
Area of a parallelogram ABCD = 2 area of Δ ABD

⎛1 ⎞ D C
A = z ⎜ x AB x AD x sinθ ⎟
⎝2 ⎠
A = AB x AD xA=
sinθ
p1
PL P2
Here = = sinθ and = sinθ
AD AD θ
A B
θ
⎛ P2 ⎞ ⎛ P1 ⎞
A =⎜ ⎟ ⎜ ⎟ x sin θ
⎝ sin θ ⎠ ⎝ sin θ ⎠ p2
p1 p2
A= sin θ

If the sides of a parallelogram be a1x+b1y+c1 = 0 ; a1x+b1y+d1 = 0 :a2x+b2y+c2 = 0, a2x+b2y+d2 =

0 area of parallelogram = (c1 – d1 )(c2 – d 2 )


a1 b1
a2 b2

Area of Rhombus
2
p1
In case of a rhombus p1 = p2, area of rhombus =
sin θ

D C D C

d1 d2 P1
P2
A B A B

1
Also area of rhombus = d d where d1, d2 are the lengths of two perpendicular diagonals
2 1 2
Area of a triangle
Area of a triangle whose vertices are (x1, y1), (x2, y2) and (x3, y3) is
1
|x (y –y ) + x2(y3–y1) + x3(y2–y1)|
2 1 2 3

12
1 x1 y1
Or
2 x2 y2
x3 y3
x1 y1

1
= [(x y +x y +x y ) – (x2y1+x3y2+x1y3)|
2 1 2 2 3 3 1
1 If area of a triangle is given then use ± sign.
2 If three points A, B, C are collinear then area of triangle ABC is zero.
3 If a1x+ b1y+c1 = 0 ; a2x+b2y+c2 = 0 and a3x+b3y+c3 = 0 are the three sides of a triangle, then
the area of the triangle is given by
2
a1 b1 c1
1
Δ = a2 b2 c2
2C1C 2 C 3
a3 b3 c3
where C1 = a2b3–a3b2
C2 = a3b1–a1b3
C3 = a1b1 –a2b1
C1,C2,C3 are cofactors of c1, c2 & c3
Area of polygon
The area of polygon whose vertices are (x1, y1), (x2y2) .............(xn1yn)

1 x1 y1 1
= = [(x1y2+x2y3.............xny1) – (y1x2+y2x3...........+ynx1)]
2 x2 y2 2
.
.
.
xn yn
x1 y1

Example
1 If the area of the rhombus by the lines x ± my ± n = 0 be 2sq. units then
(a) , m, n are in GP (b) , n, m are in GP
(c) m = n (d) L = m.
Solution : (b)
By solving the sides of the rhombus, we get the vertices of rhombus are (0, –n/m), (–n/ ,o),(0,n/m)
1 2n 2n
and (n/ ,0). Hence the area is × × =2
2 m
∴ n2 = m.

13
Hence , n, m are in GP.
2 The area of the triangular region in the first quadrant bounded on the left by the y –axis;
bounded above by the line 7x+4y = 168 and bounded below by the line 5x+3y = 121 is A, then the
y

(0,42)

121
0,
3
P(20,7)
3A 20
value of is 121
5
,0

10 0 168
x
,0
7

Solution
Given lines 7x+4y = 168 and 5x+3y = 121 intersect at P (20,7) by solving these equations
Area of shaded region is
1 ⎛ 1⎞
A= ⎜ 42 – 40 ⎟ 20
2⎝ 3⎠
1 5 50
= × × 20 = sq.units
2 3 3
3A 3 50
= × =5
10 10 3
3 Column I Column II
a Four lines x+3y–10 = 0 p a quadrilateral which is neither
x+3y–20 = 0, 3x–y+5 = 0 and parallelogram nor a trapezium
3x–y–5 = 0 form a figure which is
b The points A(1, 2), B(2,– 3), C(–1,–5) q a parallelogram
and D(–2, 4) in order are the vertices of
c The lines 7x+3y–33 = 0, 3x–7y+19 = 0, r a rectangle of area 10sq.units
3x–7y–10 = 0 and 7x+3y–4 = 0 form a
figure which is
d Four lines 4y–3x–7 = 0,3y–4x+7 = 0,
4y–3x–21 = 0,3y–4x+14 = 0 form a figure s a square
which is
Solution
a → q, r, s b →p c → q,s d →q
10 x+3y–10=0
a h1 = = 10
10
10
h2 = = 10 h1
10 3x–y+5=0
3x–y–5=0 h2

x+3y=20

14
Hence the given lines form a square of side 10 . Area of a square is ( 10 ) 2
= 10sq.units
5 D(–2,4)
b slop of AB = =–5
–1
A(1,2)
9
slop of DC = =–9
–1
O
–2 –2
slop of AD = =
3 3 B(2,–3)
C(–1,–5)
2
slop of BC =
3
Hence the figure is neither a parallelogram nor a trapezium
7x+3y–4=0
29 29
c h1 = =
58 2
h1 3x–7y+19=0
3x–7y–10=0 h2
29 29
h2 = =
58 2 7x+3y–33=0

29 29
Hence given lines form a square of side and area sq.units
2 2
d 4y–3x–21 = 0 and 4y–3x–7 = 0 are parallel and 3y–4x+14 = 0 and 3y–4x+7 = 0 are parallel.
But 4y–3x–21 = 0 and 3y–4x+14 = 0 are not perpendicular
Hcnce the given line form a parallelogram.
4y 3x 21=0

3y 4x+7=0
3y 4x+14=0

4y 3x 7=0

4 The straight lines 7x–2y+10 =0 and 7x+2y–10 = 0 form an isosceles triangle with the line y = 2.
Area of this triangle is equal to
15 10 y
(a) sq.units (b) sq.units
7 7
18 A (0,5)
(c) sq.units (d) None –6
6
,2
7 7
,2
B y=2
7

C
Solution D
x
⎛6 ⎞ ⎛–6 ⎞ 10 10
B ⎜ ,2 ⎟ and C⎜ ,2 ⎟ – ,0 ,0
7 7
⎝7 ⎠ ⎝ 7 ⎠

15
12
BC = and AD = 3
7
1 12
area of Δ ABC = × ×3
2 7
18
= sq.units.
7
Linked comprehension type
For problems 5–7
Let ABCD be a parallelogram whose equations for the diagonals AC and BD are x+2y = 3 and
2x+y = 3, respectively. If length of diagonal AC = 4 units and area of parallelogram ABCD = 8
sq.units. then
5 The length of other diagonal BD is
(a) 103 (b) 2 (c) 20/3 (d) None
6 The length of side AB is equal to
2 58 4 58 3 58 5 58
(a) (b) (c) (d)
3 9 9 9
7 The length of BC is equal to
2 10 4 10 8 10
(a) (B) (C) (d) None
3 3 3
Solution :
–1 3 x+2y=3
5 tan θ = +2 = D C
2 4
1+1
P θ
3 4 π–θ
sin θ = , cos θ =
5 5 A B
2x+y=3

1
Area of Δ CPB = PC × PB sin θ
2
1 1 3
× 8 = × 2 × PB ×
4 2 5
10
PB =
3
20
BD = units
3

16
AP 2 + PB 2 – AB2
6 cos( π – θ ) =
2AP × PB
100
4 4 + 9 – AB
2

–cos θ = =
5 10
2× 2×
3

2 58
AB =
3
PC 2 + PB2 – BC 2
7 In Δ CPB, cos θ =
2.PC.PB
2 10
BC =
3
8 ReasoningType
Statement 1 : The area of the triangle formed by the points A(1000, 1002), B(1001, 1004) C(1002,
1003) is same as the area formed by A1(0, 0), B1(1, 2), C1(2, 1)
Statement 2 : The area of the triangle is constant with respect to translation of axes.
(a) both the statements are true but statement 2 is the correct explanation of statement 1
(b) both the statement are true but statement 2 is not the correct explanation of statement1
(c) statement 1 is true and 2 is false
(d) statement 1 is false and 2 is true
Solution :
Area of triangles is unaltered by shifting origin to any point. If origin is shifted to (1000, 1002) A, B,
C become P(0, 0), Q(1, 2), R(2, 1) both are true and statement 2 is correct -explanation of state-
ment 1.
Ans. a

17
18
COORDINATE GEOMETRY-IV
Straight Line
Topics Covered
1. General Equation of straight line
2. Reducing general equations to
i. Slope intercept form
ii. Intercept form
iii. Normal form

General Equation of straight line


First degree equation of the form
ax+by+c=0 where a,b,c ∈ R a and b can be zerobut not both at the same time.
Reducing general equation to slope intercept form
Given equation is ax+by+c=0
Rewrite the equation by =-ax – c
a c
divide by b we get – x–
b b
It look like y = mx+c
−a Coefficient of x
slope = = – =m
b Coefficient of y

−c Constant
y- intereept = =–
b Coefficient of y
Reducing to intercept form
Given equation is ax+by+c=0
Rewrite the equation ax + by = −c
x y
+ =1
−c/a −c/b
x y
+ =1
a b
So intercepts are -c/a & -c/b on x-axis and y-axis respectively.

Reduce to normal form


Given equation be ax + by + c = 0
Re write the equation ax + by = – c
– ax – by = c
Keeping constant term postive
a b c
– x– y=
Divide by a 2 + b 2 we get a +b
2 2
a +b
2 2
a + b2
2

19
It look like x cosα + y sinα = p
–a –b b
Where cosα = , sin α = (or ) tan α =
a 2 + b2 a2 + b 2a
distance of a line from the origin is always positive
c
∴ = p is positive.
a 2 + b2
PRACTICE QUESTIONS

⎛ 3⎞ ⎛ 7⎞
1. A triangle ABC with vertices A(–1, 0), B ⎜ – 2, ⎟ & C ⎜ – 3,– ⎟ has its orthocenter H. Then the
⎝ 4⎠ ⎝ 6⎠
orthocentre of triangle BCH will be
(a) (–1, 0) (b) (–3, –2)
(c) (1,3) (d) (–1, 2)
2. The points A(0, 0), B(cos α , sin α ) and C(cos β , sin β ) are the vertices of a right angled triangle
if
⎛α+β⎞ 1 ⎛α+β⎞ 1 ⎛α –β⎞ 1
(a) sin ⎜ ⎟= (b) cos ⎜ ⎟ = (c) cos ⎜ ⎟ =
⎝ 2 ⎠ 2 ⎝ 2 ⎠ 2 ⎝ 2 ⎠ 2

⎛α+β⎞ 1
(d) sin ⎜ ⎟ =–
⎝ 2 ⎠ 2
3. Set of values of α for which the point ( α , α 2–2) lies inside the triangle formed by the lines x+y =
1, y = x+1 and y = –1 is
⎛ 1 – 13 ⎞ ⎛ – 1 + 13 ⎞
(a) ⎜

⎟ ⎜
⎜ 2 ,–1⎟ U⎜1,
⎠ ⎝ 2



(b) (1, 13 )
(c) (
– 13 ,–1 ) (d) None
4. Area of the parallelogram formed by the lines y = mx, y = mx+1, y = nx+1and y= nx equals
|m+n| 2 1 1
(a) (b) (c) (d)
(m – n ) 2 |m+n| |m+n| |m–n|
5. A and B are fixed points. The vertex C of Δ ABC moves such that cotA+cotB = constant. The
locus of C is a straight line
(a) perpendicular to AB (b) parallel to AB (c) inclined at an angle (A–B) to AB
(d) None of these.
6. The area of the figure formed by a|x|+b|y| +c = 0 is
c2 2c 2 c2
(a) (b) (c) (d) None of these
| ab | | ab | 2 | ab |
7. The orthocentre, circumcentre, centroid and incentre of the triangle formed by the line x+y = a with
20
the coordinate axes lie on
(a) x2+y2 = 1 (b) y = x (c) y = 2x (d) y = 3x
8. Two points (a, 3) and (5, b) are the opposite vertices of a rectangle. If the other two vertices lie on
the line y = 2x + c which passes through the point (a, b) then the value of c is
(a) –7 (b) –4 (c) 0 (d) 7.

ANSWERS
1. a 2. c 3. a 4. d 5. b 6. b
7. b 8. a

21
22
COORDINATE GEOMETRY-V
Straight Line
Topics Covered
1. Image of a point in different cases.
2. Foot of the perpendicular.
3. Family of lines.
Image of a point in different cases
i. The image of a point with respect to the line mirror
Image of A(x1, y1) with respect to line mirror ax+by+c=0 be B(x2, y2) is given by
x 2 − x1 y 2 − y1 2(ax 1 + by1 + c)
= =
a b a 2 + b2
M is the foot of prependicular from A on ax+by+c = 0

A(x1,y1)
ax+by+c=0

B(x2,y2)

ii. The image of a point with respect to x-axis:-


Let A(x1, y1) be any point and B(x2, y2) its image after reflection in the x-axis then.
x1 = x2 & y2= –y1 (M is the mid point of A&B)
y

A(x1,y1)

x
O M

B(x2,y2)

M is the foot of perpendicular from A on x-axis


iii. The image of a point with respect to y-axis:-
Let A (x1, y1) be any point and B (x2, y2) its image after reflection in the mirror y-axis then.
x2= –x1 & y1=y2 (N is the mid point of A&B)
N is the foot of perpendicular from A on y-axis
y

B(x2,y2) A(x1,y1)
N

x
O

23
iv. The image of a point with respect to the origin:-
Let A(x 1, y1) be any point B (x2, y2) be its image after reflection through the origin
then.
x2= –x1 & y2= –y1 (O is the mid point of A&B)
y

A(x 1,y1)

O x
N M

B(x2,y2)

v. The image of a point with respect to the line y = x


Let A(x1, y1) be any point and B(x2, y2) be its image after reflection in the line y=x then.
y2=x1 and x2=y1 (M is the mid point of A&B)
y
A(x1,y1)

M
B(x2 ,y2 )
O 45°
x
y=x

vi The image of a point with respect to the line y = x tan θ


Let A (x1, y1) be any point and B(x2, y2) be its image after reflection in the line
y=x tan θ or y=mx then.
x2=x1 cos2 θ +y1 sin2 θ (M is the mid point of A&B)
y2=x1 cos2 θ –y1 sin2 θ
y
A(x1,y1)

M
B(x2 ,y2 )
O θ
x
y=xtanθ

Line parallel and perpendicular to given line


Given equation of straight line be ax+by+c=0
A line parallel to given line is ax+by+d=0 only constant term changes.
A line perpendicular to given line is bx-ay+k=0
Here change coordinate of x as coordinate of y & coordinate of y as negative of coordinate
of x and constant term changes
It the lines a1x+b1y+c1 = 0 and a2x+b2y+c2 = 0 are prependecular then a1a2+b1b2= 0
a 1 b1 c1
i. Coincident, if = =
a 2 b2 c2

24
a1 b1 c1
ii. Parallel, if = ≠
a 2 b2 c2
a1 b1
iii. Intersecting, if ≠
a 2 b2

PRACTICE QUESTIONS
1. If t1 and t2 are roots of the equation t2+ λ t+1 = 0, where λ is an arbitrary constant. Then the
line joining the points (at12, 2at1) & (at22, 2at2) always passes through a fixed point
(a) (a, 0) (b) (–a, 0) (c) (0, a) (d) (0,–a)
3 3
2. The equation x +y = 0 represents
(a) three real straight lines (b) three points (c) combined equation of a st. line & a
circle (d) None of these.
3. The three lines whose combined equation is y3 – 4x2y = 0 form a triangle which is
(a) isosceles (b) equilateral (c) right angled (d) None of these
Comprehension Type
⎛ 2 2⎞
A(1, 3) and C ⎜ – ,– ⎟ are the vertices of a triangle ABC and the equation of the angle bisector
⎝ 5 5⎠
of ∠ ABC is x+y = 2
4 Equation of side BC is
(a) 7x+3y = 4 (b) 7x+3y+4 = 0 (c) 7x–3y+ 4 = 0 (d) 7x–3y = 4
5. Coordinates of vertex B are
⎛ 3 17 ⎞ ⎛ 17 3 ⎞ ⎛ 5 9⎞
(a) ⎜ , ⎟ (b) ⎜ , ⎟ (c) ⎜– , ⎟ (d) (1, 1)
⎝ 10 10 ⎠ ⎝ 10 10 ⎠ ⎝ 2 2⎠
6. Equation of side AB is
(a) 3x+7y=24 (b) 3x+7y+24 = 0 (c) 13x+7y+8 = 0 (d) 13x–7y+8 = 0
7. Assertion and reasoning Type
Lines L1 L2 given by y –x = 0 and 2x + y = 0 intersect the line L3 given by y+2 = 0 at P and Q,
respectively. The bisector of the acute angle between L1 and L2 intersects L3 at R.
Statement 1 : The ratio PR : RQ equals 2 2 : 5 .
Statement 2 : In any triangle, bisector of an angle divides the triangle into two similar triangles.
a Statement 1 is true, Statement 2 is True; Statement 2 is a correct explanation for State-
ment1.
b Statement 1 is True, Statement 2 is True; Statement 2 is NOT a correct explanation for
Statement 1.
c Statement 1 is True, Statement 2 is False.
d Statement 1 is False, Statement 2 is True.
8. Matrix-match
This question contains statements given in two columns which have to be matched. Statements a,
b, c, d in column I have to be matched with statements p,q, r, s in column II. If the correct match
is a → p, a → s, b → q b → r, c → p,c → q and d → s, then the correctly dubbled 4×4 matrix

25
should be as follows :
Consider the lines given by
L1 : x + 3y – 5 = 0
L2 : 3x – ky –1 = 0
L3 : 5x + 2y –12 = 0

Column I Column II
a L1, L2, L3 are concurrent, if p. k=–9
b One of L1, L2, L3 is parallel to at least one of
the other two, if q. k = – 6/5
c L1, L2, L3 form a triangle, if r. k = 5/6
d L1, L2, L3 do not form a triangle, if s. k=5

ANSWERS
1 b 2 d 3 d 4 b 5 c 6 a
a→s c→r
7 c 8 b → p, q d → p, q, s

26
COORDINATE GEOMETRY-VI
Straight Line
Topics Covered
1. Concurrent lines.
2. Equations of angles bisectors.
3. Position of two points relative to a given line.
4. Standard points of triangle.
5. Equations of straight lines through (x1, y1) making angle α with line y=mx+c

1. Concurrent lines
The three given lines are concurrent if they meet at one point.
i. Find the point of intersection of any two lines by solving them simultaneously. If this
point satisfies the third equation also then the given lines are concurrent.
ii. Let three lines be
L1=a1x+b1y+c1= 0 L2=a2x+b2y+c2=0 and
L3=a3x+b3y+c3= 0 are concurrent if
a 1 b1 c1
a 2 b 2 c 2 = 0
a 3 b 3 c 3
iii. The three lines L1=0, L2=0 and L3=0 are concurrent if there exist constants , m and n
not all zero at the same time, such that
L1+mL2+nL3= 0
2. Eqnations of angle bisectors between two lines
The eqnations of the bisectors of the angles between the lines a1x+b1y+c1= 0 and a2x+b2y+c2=0
a1x + b1 y + c1 a 2x + b2 y + c2
are given by =±
a +b
2
1
2
1 a 22 + b 22
i. Any point on a bisector is equidistant from the given lines.
ii. Locus of points which are equidistant from the two intersecting lines is an angle bisector.
iii. Bisectors are perpendicular to each other.
iv. Equation of the bisector of the acute and of obtuse angle between two lines.
Let a2x+b2y+c2= 0 ......(1)
and a2+b2y+c2=0 ......(2)
c1>0,c2>0 then the equation
a1 x + b1 y + c1 a 2x + b2 y + c2
=+
a +b
2
1
2
1 a 22 + b 22
is the bisector of the acute or obtuse angle between the lines 1&2 according as
a1a2+b1b2 < 0 or > 0
a 1 x + b1 y + c1 a 2x + b2 y + c2
again =–
a 12 + b12 a 22 + b 22
27
is the bisector of the acute or obtuse angle between the lines 1&2 according as
a1a2+b1b2 > 0 or < 0

3. Position of two points relative to a given line


Let the line be L =ax+by+c=0
P(x1,y1) and A(x2,y2) are two given points.
ax1 + by1 + c
i. If ax1+by1+c and ax2+by2+c both are of the same sign and hence ax + by + c >0 then
2 2

the points P and Q lie on same side of line ax+by+c=0


L L

P
Q P
Q

ax1 + by1 + c
ii. If ax1+by1+c and ax2+by2+c are of opposite sign and hence ax + by + c <0 then the
2 2

points P and Q lie on opposite side of the line ax+by+c=0


L

iii. If origin lie on line then the line is know as origin side.
L L
(0, 0)

O(0, 0)

P(x1,y1 )

P(x1,y1)

iv. A point (x1y1) will lie on origin side of the line ax+by+c=0 if ax1+by1+c and c have same
sign.
v. A point (x1, y1) will lie on non-origin side of the line ax+by+c=0 if ax1+by1+c and c have
opposite sign

Family of straight lines


Let L1 = a1x+b1y+c1=0 and L2 =a2x+b2y+c2=0
Then the general equation of any straight line passing through the point of intersection of lines L1
and L2 is given by L1+ λ L2=0 where λ is any real number.

Equations of straight lines through (x1,y1) making angle α with y=mx+c


m + tanα m – tanα
y-y1= (x − x1)and y – y1 = (x − x1)
1− mtanα 1+ mtanα

28
L1
θ
α
L2

α θ–α
θ

Standard points of a triangle


1. Centroid or centre of gravity :- The centroid of a triangle is the point of intersection of its
median’s. the centroid divides the median in the ratio 2:1 (vertex:base).
x1 + x 2 + x 3 y1 + y 2 + y 3
Coordinates of G are G ,
3 3

A(x1,y1)

2
E F
G

C(x3,y3) D B(x2,y2)

In isosceles triangle median to the equal sides are equal in length and in equilateral triangle all
medians are equal in length.
Equations of median can be obtained by using two point form with vertex and the mid point of
opposite side.

2. Circumcentre:- The circumcentre of a triangle is the point of intersection of the perpendicular


bisectors of the sides of a triangle.
Coordinates of O can be obtained from the equation.
OA2 =OB2=OC 2
A(x 1,y 1)

C(x 3,y 3) B(x 2,y 2)

If angles A,B,C and vertices A(x1,y1) , B(x2,y2) and C(x3,y3) of a ΔABC are given; Then its
x1 sin 2A + x 2 sin 2B + x 3 sin 2C y1 sin 2A + y 2 sin 2B + y 3 sin 2C
circumcentre is given by ,
sin 2A + sin 2B + sin 2C sin 2A + sin 2B + sin 2C
The circumcentre of a right-angled triangle is the mid-point of its hypotenuse. Therefore the
mid-point of hypotenuse is equidstant from its vertices
AM=BM=CM

29
A

B C

The circumcentre of the triangle formed by (0,0), (x1,y1) and (x2,y2) is


( ) ( ) ( ) (
y 2 x12 + y12 − y1 x 22 + y 22 x 2 x12 + y12 − x1 x 22 + y 22
,
)
2(x1 y 2 − x 2 y1 ) 2(x 2 y1 − x1 y 2 )

3. Incentre of a triangle:- The point of intersection of the internal bisecters of the angles of a
triangle is called the incentre of the triangle.
The coordinates of the incentre of a triangle with vertices (x1, y1), (x2, y2) and (x3, y3) are
ax1 + bx 2 + cx 3 ay1 + by 2 + cy 3
,
a+b+c a+b+c
Where a,b,c are lengths of sides of triangle
A

c b
I

B C
a

ab ab
The incentre of the triangle formed by (0,0), (a,0), (0,b) is ,
a + b + a + b a + b + a2 + b2
2 2

4. Orthotcentre:- The orthocentre of a triangle is a point intersection of altitudes.


i. Take the equations of any two sides of a triangle. find the eqnations the lines
perpendicular to those lines and passing through the opposite vertices. solve these two
equations we get orthocentre of the triangle.
ii. If angles A,B,C and vertices A (x 1, y 1) B (x 2, y2) and C (x3, y 3) of a ΔABC are given
then orthocentre of ΔABC is given by
x1 tan A + x 2 tan B + x 3 tan C y1 tan A + y 2 tan B + y 3 tan C
,
tan A + tan B + tan C tan A + tan B + tan C

iii. If any two lines out of three lines AB, BC, CA, are perpendicular, then orthocentre is the
point of intersection of two perpendicular lines.
iv. The orthocentre of the triangle with verties (0,0), (x1, y1) and (x2, y2) is

30
x1x 2 − y1 y 2 x x + y1 y 2
(y1 − y 2 ) ,(x1 − x 2 ) 1 2
x 2 y1 − x1 y 2 x1 y 2 − x 2 y1
v. The orthocentre (O), centroid (G) and circumcentre (C) of any triangle lie in a straight
line and G divides the join of O and C in the Ratio 2 :1
vi. In an equilateral triangle,orthocentre, centroid, circumcentre and incentre coinside.

5. Coordinates of nine point circle :- If a circle passes through the feet of perpendicular (D,E,F)
mid points of sides BC, CA, AB sespectivcly (H,I,J) and mid points of the line joining the
orthocentre O to the angular points A,B,C (K,L,M) thus the nine points D,E,F H,I,J, K,L,M all
lie on a circle, This circle is known as nine point circle and its centre is called nine point centre.
A

E
K

F I
J

M
L

B D C
H

1. The orthocentre (O), nine point centre (N) centroid (G) and circumcentre (C) all lie in the
same line i.e. ONGC (oil natural gas corporation)
2. The nine point centre bisects the join of orthocentre (O) and circumcentre (C)
3. The radius of nine point circle is half the radius of circumcircle.

31
32
COORDINATE GEOMETRY-VII
Straight Line
Examples
1. Locus of the image of the point (2,3) in the line (x-2y+3)+λ(2x-3y+4)=0 is
(a) x2+y2-3x-4y-4=0
(b) 2x2+2y2+2x+4y-7=0
(c) x2+y2-2x-4y+4=0
(d) None
x − 2y + 3 = 0
Solution: P (1,2)
2 x − 3y + 4 = 0
Let image be B(h,k) then AP=BP
(h-1)2+(k-2)2 =12+12
h2+k2-2h-4k+5=2
x2+y2-2x-4y+3=0

P B
A(2,3)
(h,k)
(1,2)

Ans. : d
2. If the lines ax+y+1=0, x+by+1=0 and x+y+c=0 (a,b,c being distinct and different from 1) are
⎛ 1 ⎞ 1 1
concurrent, then ⎜ ⎟+ + =
⎝1− a ⎠ 1− b 1− c
1
(a) 0 (b) 1 (c) (d) None
a+b+c

Solution:
c2→c2–c1,c3→c3–c1
a 1 1 a 1– a 1– a
1 b 1 = 0 ⇒ 1 b –1 0 = 0
1 1 c 1 0 c –1
a(b-1)(c-1)-(1-a)(c-1)+(1-a)(1-b)=0
divide by (1-a)(1-b)(1-c) we get
a 1 1
⇒ + + =0
1− a 1− b 1− c
1 1 1
⇒ + + =1
1− a 1− b 1− c
33
3. The set of values of ‘b’ for which the origin and the point (1,1) lie on the same side of the st. line
a2x+aby+1=0 ∀ a ∈R, b>o are
(a) b∈[2,4) (b) b∈(0,2) (c) b∈[0,2] (d) None of these

Solution: (0,0) & (1,1) are on the same side


0+1>0 so a2+ab+1>0
b2 − 4 < 0
⇒ D<0 i.e
b2 < 4
-2<b<2 but b>0
∴ b∈(0,2)

4. ( )
Let P(-1,0), Q(0,0) and R 3,3 3 be three points. Then the equation of the bisector of the
angle PQR is

3 3
(a) x+y=0 (b) x + 3y = 0 (c) 3x + y = 0 (d) x+ y=0
2 2
Solution: 3−m m
=
1 + 3m 1+ 0
3 − m = m + 3m 2 ⇒ 3m 2 + 2m − 3 = 0
3m 2 + 3m − m − 3 = 0
( )(
3m − 1 3 + 3 = 0 )
1
m= ,− 3
3
y = − 3x
3x + y = 0
R
m
m= 3

P m=0 Q
5. OPQR is a square and M, N are the mid points of the sides PQ and QR respedively. If the ratio
λ
of the areas of the square and the triangle OMN is λ : 6, then is equal to
4
(a) 2 (b) 4 (c) 12 (d) 16

34
Solution: ar. of square = a2

0 0 1
a 1 3a 2 3a 2
1 a 1= . =
ar of Δ OMN = 2 2 4 8
2 a
a 1
2

a2 λ
2
= ⇒ λ = 16
3a 6
8
Ans : b
Y a
N ,a
2
R Q

a
M a,
2

X
O a P

6. A pair of perpendicular straight lines drawn through the origin form an inosceles triangle with
line 2x+3y=6, then area of the triangle so formed is
36 12 13 17
(a) (b) (c) (d)
13 17 5 13

−6 6 6 1 2×6 6 36
Solution: OM= = , PQ= 2 × , area od ΔOPQ = × × =
4+9 13 13 2 13 13 13
Q

45
Q M
M
P
2x+ 45 P
3y-
(0,0) 6=0
o 45
45

35
7. Match the column
Column I Column II
(a) Two vertices of a triangle are (5,–1) and (–2,3) if orthocentre (p) (-4,-7)
is origin then third vertex is
(b) A point on the line x+y=4 which lies at a unit distance from (q) (-7,11)
the line 4x+3y=10, is
(c) Orthocentre of the triangle made by the (r) (1,-2)
lines x+y-1=0, x-y+3=0, 2x+y=7 is
(d) If a,b,c are in A.P., then lines ax+by=c always Pass through (s) (-1,2)

Solution: A-p
B-q
C-s
D-s

36
COORDINATE GEOMETRY-VIII
Straight Line
Examples
1. The number of integer values of m for which the x- coordinate of the point of intersection of the lines
3x+4y=9 and y=mx+1 is also an integer is
(a) 2 (b) 0 (c) 4 (d) 1
⎛ 5 3 + 9m ⎞
Solution: Point of intersection ⎜ , ⎟
⎝ 3 + 4m 3 + 4m ⎠
x-coordinate is an integer when 3+4m = ±1 or ±5.
1 1
m = -1,-2, − ,
2 2
Hence there are two values of m.
2. A straight line through the origin meets the parallel lines 4x+2y=9 and 2x+y+6=0 at points P
& Q respectively. Then the point O divides the segment PQ in the ratio
(a) 1:2 (b) 3:4 (c) 2:1 (d) 4:3
Solution: Clearly Δ OPA~ Δ OQC.
9
OP OA 3
⇒ = = 4=
OQ OC 3 4
3:4

⎛ 9⎞
B⎜ 0, ⎟
⎝ 2⎠

C(-3,0) o
⎛9 ⎞
Q A⎜ ,0 ⎟
⎝4 ⎠
D(0,-6)

3. A family of lines is given by (1+2λ)x+(1-λ)y+λ=0, λ being the parameter. The line belonging
to this family at the maximum distance from the pt (1,4) is
(a) 4x-y+1=0 (b) 33x+12y+7=0 (c) 12x+33y=7 (d) None of these.
Solution: x+y+λ(2x-y+1)=0
1
4−
y− =
1 3 ⎛x + 1⎞ ⇒
⎜ ⎟
The required line is y 3 ⎠ 12x+33y=7.
1+ ⎝
3 1
3
P(1,4)
x+y=0

⎛ 1 1⎞
⎜− , ⎟
⎝ 3 3⎠ 2x-y+1=0
37
4. The four sides of a quadrilateral are given by the equation xy(x–2)(y-3)=0. The equation of the line
parallel to x-4y=0 that divides the quadrilateral in two equal areas is
(a) x-4y+5=0 (b) x-4y-5=0 (c) 4y=x+1 (d) 4y+1=x.

1
Solution: ar.of OADE= ar OABC
2
1 1
(OE+AD)×OA = ×OA×AB
2 2
1⎛ λ 2–λ⎞ 1
⎜− + ⎟ ×2 = ×2×3
2⎝ 4 4 ⎠ 2
2 − 2λ
=3
4
∴ λ =-5
∴ Equation is x-4y+5=0

C B
x-4y=λ
D
E
x-4y=0

O A

5. Consider the points A(0,1) and B(2,0) and P be a point on the line 4x+3y+9=0. Coordinates of
P such that |PA–PB| is maximum are
⎛ − 24 17 ⎞ ⎛ − 18 13 ⎞ ⎛ 31 31 ⎞
(a) ⎜
⎝ 5 5⎠
, ⎟ (b) ⎜
⎝ 5 5⎠
, ⎟ (c) ⎜ , ⎟
⎝7 7⎠
(d) (0,0)
Solution: |PA-PB|≤AB
0 −1
Thus |PA-PB| is max. if points A,B,P are collinear. Equation of AB is y-1= (x-0)⇒x+2y-
2 −1
2=0
⎛ − 24 17 ⎞
Hence solving x+2y-2=0 & 4x+3y+9=0 we get ⎜ , ⎟
⎝ 5 5⎠

38
P
A(0, 1)
P1

B(2, 0)

4x+
3y+
9=0

6. A light ray coming along the line 3x+4y=5 gets reflected from the line ax+by=1 and goes
along the line 5x-12y=10. Then
64 112 14 −8 64 −8
(a) a= ,b = (b) a= ,b = (c) a = ,b =
115 15 15 115 115 115
64 14
(d) a= ,b =
15 15

Solution: ax+by=1 will be one of the bisectors of the lines given


3x + 4 y − 5 ⎛ 5x − 12 y − 10 ⎞
= +⎜ ⎟
5 ⎝ 13 ⎠
⇒ 64x-8y=115 or 14x+112y=15 On comparing with ax+by=1, we get
64 −8 14 112
a= ,b = or a= ,b =
115 115 15 15

Ans: (C)

7. If the point (a,a) is placed in between the lines |x+y|=4, then a is


(a) [-2,0] (b) [0,2] (c) (-2,2) (d) [-2,2]

Solution: x+y=4
x+y=-4
& pt (2,2) & (-2,-2) lies on these lines
Then pt. (a,a) lies between the lines then a>-2 and a<2 ie. -2<a<2.
Ans: (C)
39
y=x

(2,2)

(-2,-2)

x+y=2

x+y=-2

8. Consider the family of lines 5x+3y-2+λ1(3x-y-4)=0 and x-y+1+λ2(2x-y-2)=0. The equation


of a straight line that belongs to both the families is _______________________.
Solution: Equation of a line belongs to both families
passes through A and B is

5 x + 3y = 2 4 +1
x − y = −1 y +1 = (x − 1)
3x − y = 4 3 −1
2x − y = 2
5
x =1 x=3 y + 1 = (x − 1)
y = −1 2
y=4 5x − 2 y − 7 = 0

A B
(1,–1) (3,4)

9. If x1, x2, x3 as well as y1, y2, y3 are in G.P. with same common ratio then the points P(x1,
y1),Q(x2,y2) and R (x3,y3)
(a) lie on a straight line
(b) lie on an ellipse
(c) lie on a circle
(d) are vertices of a triangle.

Solution: Let x1=a, x2=ar, x3=ar2


y1=b, y2=br, y3=br2

40
y 2 − y1 b y − y2 b
Now = & 3 =
x 2 − x1 a x3 − x2 a
slope of PQ = slope of QR
Hence pts P,Q,R are collinear.
Ans. : a
x y
10. A variable straight line is drawn through the point of intersection of the straight lines + =1
a b
x y
and + = 1 and meets the coordinate axes at A and B, find the locus of the midpoint of AB.
b a
Solution: Let mid pt. of AB be (h,k).
⎛ ab ab ⎞
Intersection pt. of given lines is ⎜ , ⎟ B
⎝a +b a +b⎠
P is mid pt. of AB ⇒ A(2h,0) & B(0,2k).
P(h,k)
(0, b)

2h 0 1
(0, a)
0 2k 1 = 0 Q
Now A,B and Q are collinear ⇒ ab ab
1 A
a+b a+b (a, 0) (b, 0)
2hab 2kab
⇒ 4hk – – =0
a+b a+b
⇒ 2xy(a+b) = ab(x+y).
PRACTICE QUESTIONS
1. If the distance of any point (x,y) from origin is defined as d(x, y) =⏐x⏐+⏐y⏐ then the locus of d(x,
y)=1 is a
a. circle of area π sq. units b. square of area 1 sq. units
c. square of area 2 sq. units d. none of the above
π
2. The line 3x–4y+7 = 0 is rotated through an angle in the clockwise direction about the point
4
(–1, 1). The equation of the line in its new position is
a. 7y+x–6=0 b. 7y–x–6=0 c. 7y+x+6=0 d. 7y–x+6=0
3. The number of integral values of m for which the x-coordinate of the point of intersection of
the lines 3x +4y = 9 and y = mx +1 is also an integer is
a. 2 b. 0 c. 4 d. 1
4. One of the bisector of the angle between the lines a(x–1) +2h(x–1)(y–1) + b(y–2)2=0 is x+2y–
2

5=0 The other bisector


a. 2x–y=0 b. 2x+y=0 c. 2x+y–4=0 d. x–2y+3=0
5. The point A(2, 1) is translated parallel to the line x–y=3 by a distance 4 units. If the new
position A is in third quadrant, then the coordinates of A’ are

41
a. (2+2 2 , 1+2 2 ) b. (–2+ 2 , –1–2 2 )
c. (2–2 2 , 1–2 2 ) d. none
6. The orthocenter of the triangle formed by the lines x+y=1, 2x+3y=6 and4x–y+4=0 lies in
a. I quadrant b. II quadrant c. III quadrant d. IV quadrant
7. The reflection of the point (4, –13) in the line 5x+y+6=0 is
a. (–1, –14) b. (3, 4) c. (1, 2) d. (–4, 13)
8. If p1, p2, p3 be the length perpendiculars from the points (m2, 2m), (mm1, m+m1) and (m12, 2m1)
sin 2 α
respectively on the line xcosα + ysinα + =0 then p1, p2, p3 are in
cos α
a. AP b. GP c. HP d. none of these
9. If f(+y) =f(x)+f(y), ∀ x, y ε R and f(1)=2, then area enclosed by 3⏐x⏐+2⏐y⏐≤8 is
⎛1⎞ 1 1
a. f(4) sq. u b.⎜ ⎟ f(16) sq. u c. f(6) sq. u d. f(5) sq. u
⎝2⎠ 3 3
10. The medians AD and BE of the triangle with vertices A(o, b), B(o, o) and C(a, o) are mutually
perpendicular if
a. b= 2 a b. a = 2b c. b=– 2 a d. a=5 2 b
11. The graph of the function y=cosxcos(x+2)–cos2(x+1) is
a. a straight line passing through (0, –sin21) with slope 2.
b. a straight line passing through (0, 0)
c. a parabola with vertex (1, –sin21)
⎛π 2 ⎞
d. a straight line passing through the point ⎜ ,– sin 1⎟ parallel to the x-axis.
⎝2 ⎠
12. If line 2x+7y–1=0 intersect the lines L1=3x+4y+1=0 L2=6x+8y–3=0 in A and B respectively,
then equation of a line parallel to L1 and L2 and passes through a point P such that AP:PB=2:1
(internally) is (P is on the line 2x+7y–1=0).
a. 9x+12y+3=0 b. 9x+12y–3=0 c. 9x+12y–2=0 d. none of these
13. The equation of a line through the point (1, 2) whose distance from the point (3, 1) has the
greatest possible value is
a. y=x b. y=2x c. y=–2x d. y=–x
14. If the point (cosθ, sinθ) does not fall in that angle between the lines y=⏐x–1⏐ in which the
origin lies, the θ belongs to
⎛ π 3π ⎞ ⎛ π π⎞
a. ⎜ , ⎟ b. ⎜– , ⎟ c. (0, π) d. none of these
⎝2 2 ⎠ ⎝ 2 2⎠
15. A line of fixed length (a+b) moves so that its ends are always on two fixed perpendicular
lines. The locus of the point which divides this line into portion of lengths a and b is
a. a circle b. an ellipse c. a hyper bola d. none of these
Numerical Grid Based problems
Solve the following problems and mark your response against their respective grids. Write
your answer in the top row of the grid and darken the concerned numbers in the respective
columns.
42
16. The locus of the moving point P such that 2PA=3PB Where A is (0, 0) and B is (4, –3) is 5x2+5y2–
72x+λy+μ=0 then the value of λ+μ must be.
17. If (λ,2) is an interior point of ΔABC formed by x+y=4, 3x–7y=8 and 4x–y=31 then λε(a, b), the
value of 6a+8b must be.
x y x y
18. A variable straight line through the point of intersection of the lines + =1 and + =1
3 4 4 3
meets the coordinate axes in A and B. If locus of the mid point of AB is 2xy=k(x+y) then the
value of 343 k must be

Assertion and Reason (for θ 9-21)


a. If Assertion is true and reasoing is also true,α and reasoning is a correct explanation for assertion.
b. If Assertion is True Reasning is also true but reasoing is not correct explanation for Assertion.
c. Assertion is true reasoning is false Assertion is false is true.
19. Assertion (A) : If centroid and circumcenter by a triangle are known, then its orthocenter and nine
point centre can be found.
Reason (R) : Orthocenter, nine point centre, centroid and Circumcenter are collinear.
a. A b. B c. C d. D
20. Assertion (A) : The line 2x+y+6=0 is perpendicular to the line x–2y+5=0 and second line
passes through (1, 3)
Reason (R) : Product of the slopes of the lines is equal to –1.
a. A b. B c. C d. D
⎛π⎞ ⎛π⎞
21. Assertion (A) : The incenter of the triangle formed by the lines xcos ⎜ ⎟ +ysin ⎜ ⎟ –π=0, x
⎝9⎠ ⎝9⎠
⎛π⎞ ⎛π⎞ ⎛ 4π ⎞ ⎛ 4π ⎞
cos ⎜ ⎟ –ysin ⎜ ⎟ +π=0 and x cos ⎜ ⎟ + y sin ⎜ ⎟ +π=0 is (0, 0)
⎝9⎠ ⎝9⎠ ⎝ 9 ⎠ ⎝ 9 ⎠
Reason (R) : The point (0, 0) is equidistant from the three vertices of the triangle formed by
π π π π ⎛ 4π ⎞ ⎛ 4π ⎞
the lines xcos +ysin –π=0, x cos –ysin +π=0, xcos ⎜ ⎟ +ysin ⎜ ⎟ +π=0
9 9 9 9 ⎝ 9 ⎠ ⎝ 9 ⎠
a. A b. B c. C d. D
ANSWERS
1. c 2. a 3. a 4. a 5. c 6. a
7. a 8. b 9. c 10. b 11. d 12. c
13. b 14. b 15. a 16. λ=54 μ=225, λ+μ =54+225=279
22 33 22 33 12 12
17. <λ< a= ,b= 18. Κ= 343Κ = 343 × =49 ×12=588
3 4 3 4 7 7
22 33
[6a + 8b = 6 × +8× =44+66]
3 4
= 44 + 66
= 110
19. d 20. a 21. b
43
44
CIRCLE - I
Equation of circles
BASIC CONCEPTS
P
Circle : A circle is the locus of points which are equidistant from a fixed point and
lies on the same plane. r

Fixed point is called centre of a circle and constant distance is called radius of the o
circle
STANDARD EQUATION OF A CIRCLE
The equation of a circle with the centre at (h,k) and radiaus r is
(x–h)2 + (y–k)2 = r2 P(x,y)
r
If centre is at the origin and radius is r then the equation of circle is x2+y2
=r2 c(h,k)
GENERAL EQUATION OF A CIRCLE
x2+y2+2gx+2fy+c = 0 where g, f, and c are constants

centre (–g,–f) and radius is g 2 + f 2 – c

CONDITIONS FOR A SECOND DEGREE EQUATION TO REPRESENT A CIRCLE


ax2+2hxy+by2+2gx+2fy+c = 0 is a second degree equation
(i) coefficient of x2 = coefficient of y2. ie., a=b
(ii) coefficient of xy = 0 ie., h=0
If g2+f2–c>0 then the circle represents real circle with centre (–g, –f)
If g2 +f2 – c = 0 then the circle represents point circle since radius is zero
If g2+f2–c<0 then the circle is imaginary circle .
EQUATION OF CIRCLE IN VARIOUS FORMS
1 Equation of circle with centre (h.k) and passes through origin. is x2+y2+2hx+2ky = 0
Note that when a circle passes through origin the constant term must be zero

C(h,k)

o x

45
2 If the circle touches x–axis then its equation is (x ± h)2 +(y ± k)2 = k2 (or) x2+y2 ± 2hx ± 2ky+h2=0.
In this case radius is ordinate of centre of a circle. Four circles possible

o x

3 If the circle touches y–axis then its equation is (x ± h)2 + (y ± k)2 = h2 (or) x2+y2 ± 2hx ± 2ky+k2
=0. Here radius of the circle is abscissa of the centre. Four circles possible.
y

o x

4 If the circle touches both the axes then its equation is (x ± r)2+(y ± r)2 =r2. Four circles possible
x2+y2 ± 2rx ± 2ry+r2 = 0
y

o x

5 If the circle touches x–axis at origin then its equation is x2+(y ± k)2 = k2
x2+y2 ± 2ky = 0

46
6 If the circle touches y–axis at origin then its equation is (x ± h)2+y2 = h2 (or) x2+y2 ± 2hx = 0
y

o x

7 If the circle passes through origin and cuts intercepts a and b on the axes, then the equation of
circle is x2+y2–ax–by = 0 and centre is c(a/2, b/2) four circles possible.
y

(a/2, b/2)
b

x
a

8 Equation of Circle on a given Diameter


If (x1,y1) and (x2,y2) are end points of the diameter then the equation of circle is
(x–x1)(x–x2) + (y–y1)(y–y2) = 0
P(x,y)

A B
(x1,y1) C (x2,y2)

9 Parametric form of Circle


x = h+rcos θ
y = k+rsin θ
Where θ is parameter (0 ≤ θ ≤ 2 π )
In particular coordinates of any point on the circle x2+y2 = r2 is (rcos θ , rsin θ ) on the circle

(
x2+y2+2gx+2fy+c =0 is – g + g 2 + f 2 – c (cos θ),–ƒ + g 2 + f 2 – c (sin θ) )
y
P(x,y)
r
θ
C(h,k)

x
o

47
10 Intercept made on the axes by a circle y
Let the equation of circle is x2+y2+2gx+2fy+c = 0 D

(-g,-f)
AB = x - intercept = 2 g 2 – c N

0 A M B x
CD = y - intercept = 2 f – c
2

48
CIRCLE - II
Position of a Point
P
1. Position of a point with respect to a circle . P
let the circle is x2+y2+2gx+2fy+c = 0
P(x1,y1)
Point P(x1, y1) lies outside,on or inside the circle C
accordingly CP>, = , < radius
(or) S1 = x12+y12+2gx1+2fy1+c>, = , < 0
2 Maximum and Minimum Distance of a point from the circle
Let the circle x2+y2+2gx+2fy+c = 0 and point P(x1, y1)
The maximum and minimum distance form P(x1, y1) to the
circle are
PB = CB +CP B P(x ,y )
C A 1 1

= r + CP
PA = |CP–CA| = |PC–r|
PB is maximum distance and PA is minimum distance.
EXAMPLES
1 If the equation px2+(3–q) xy +2y2–6q x+30y+6q = 0 represents a circle, then the values of p and
q are
(a) 3,1 (b) 2,2 (c) 2,3 (d) 3,4
Solution :
ax2+by2+2hxy+2gx+2fy+c 0
represents a circle if h = 0 and a = b
∴ p = 2 and 3 – q = 0 ⇒ q = 3
correct option is (c)
2 The number if integral values of λ for which x2+y2+(1– λ )x+ λ y+5 =0 is the equation of a circle
whose radius cannot exceed 5 is
(a) 20 (b) 16 (c) 18 (d) 24
Solution :

radius of the equation g 2 + f 2 – c


1– λ λ
g= , f= c=5
2 2
2 2
⎛1 – λ ⎞ λ
⎜ ⎟ + – 5≤ 5
⎝ 2 ⎠ 4
1+ λ 2–2 λ + λ 2–20 ≤ 100
2 λ 2–2 λ –119 ≤ 0

49
D = 4+8×119
= 4 +952
= 956
= 4(239)

2 ± 2 239
λ =
4

1– 239 1 + 239
=
2 2

1– 239 1 + 239
∴ ≤ λ≤
2 2
–7.2 ≤ λ ≤ 8.2 (approx)
λ = –7, –6, –5,–4, ........ 8
number of values of λ is 16
3 The equation of the circle which passes through (1, 0) and (0, 1) and has its radius as small as
possible is
(a) x2+y2–2x–2y+1 = 0 (b) x2+y2–x–y = 0
(c) 2x2+2y2–3x–3y+1 = 0 (d) x2+y2–3x–3y+2 = 0
Solution :
The radius will be minimum, if the given points are the end points of a diameter. Then the equation
of the circle is (x–1) (x–0) + (y–0)(y–1) = 0 ⇒ x2+y2–x–y = 0
4 The centre of the circle x = –1+ 2cos θ , y = 3+2sin θ is
(a) (1,–3) (b) (–1, 3) (c) (1, 3) (d) None of these
Solution :
Rewrite the given equation
x+1 =2cos θ y–3 = 2sin θ
x +1 y–3
=cos θ = sin θ
2 2
squaring and adding
2 2
⎛ x +1⎞ ⎛ y – 3⎞
⎜ ⎟ +⎜ ⎟ = cos2 θ + sin2 θ
⎝ 2 ⎠ ⎝ 2 ⎠
(x+1)2 + (y–3)2 = 4
centre (–1, 3) radius 2.

50
CIRCLE - III
Problem Solving
1. The number of points (x,y) having integral coordinates satisfying the condition x2+y2<25 is
(a) 90 (b) 81 (c) 80 (d) 69
Solution :
Since x2+y2<25 and x and y are integers, the possible values of x & y ∈ (0, ± 1, ± 2, ± 3, ± 4),
thus x and y can be chosen in 9 ways each and (x1 y) can be 9×9 = 81 ways. But ( ± 3, ± 4)
( ± 4, ± 3) ( ± 4, ± 4) does not satisfy so we must exclude these points 3×4 = 12 ways.
Hence the number of permissible values are 81–12 = 69.
2. A point P moves in such a way that the ratio of its distance form two coplanar points is always a
fixed number ( ≠ 1) then its locus is
(a) Straight line (b) circle (c) Parabola (d) a pair of Straight lines
Solution :
Let two coplanar points be (0, 0) and (a, 0) according to the question we get
AP
= λ (constant)
BP

x 2 + y2

(x – a )2 + y 2
x2+y2 = λ 2(x2+y2–2ax+a2)

2 2
λ2
x +y + (2ax–a2) = 0
1 – λ2
represents equation of a circle
3. The greatest distance of the point P (10, 7) from the circle
x2+y2–4x–2y–20 = 0 is
(a) 10 (b) 15 (c) 5 (d) None of these
Solution :
Given equation of circle in x2+y2–4x–2y–20=0
S1 = 102+72–40–14–20>0
∴ P(10, 7) lies outside the circle.
PB = PC+CB

= r + 82 + 6 2 B P(10,7)
C (2,1) A

= r +10
= 4 + 1 + 20 + 10
= 5+10
= 15

51
4. If one end of a diameter of the circle x2+y2–4x–6y+11 = 0 be (3, 4), then the other end is
(a) (0, 0) (b) (1, 1) (c) (1,2) (d) (2,1).
Solution :
Centre of the circle is (2,3) one end of the diameter
is (3,4). Since centre is the mid point of diameter
B A
C (2,3)
α+3 β+4 , β)

∴ =2 & =3
2 2
α = 1, β = 2
∴ Other end points is (1,2).
5. ƒ(x,y) = x2+y2+2ax+2by+c = 0 represents a circle . If ƒ(x,0) = 0 has equal roots each being 2 and
ƒ(0,y) = 0 has 2 and 3 as its roots then the centre of circle is
(a) (2,5/2) (b) date are not sufficient
(c) (–2,–5/2) (d) data are inconsistent
Solution :
ƒ(x,0) = 0 ⇒ x2+2ax+c = 0 = (x–2)2
∴ c = 4. & 2a = –4. ⇒ a = – 2
ƒ(0,y) = 0 ⇒ y2+2by+c = 0 = (y–2) (y–3)
= y2–5y+6
∴ b = –5/2 & c = 6
∴ c is not unique so data are inconsistent
6. If p and q are the largest distance and the shortest distance respectively of the point (–7,2) from
any point ( α , β ) on the curve whose equation is x2+y2–10x–14y–51 = 0 then G.M of p and q is
equal to
(a) 2 11 (b) 5 5 (c) 13 (d) None of these
Solution :
The centre C of the circle is (5,7) and the radius is 25 + 49 + 51 = 125 = 5 5

PC = 122 + 52 = 169 = 13
∴ p = PB = PC+CB q = PA = PC–CA P(2,4)

= 13+ 5 5 = 13– 5 5
A
P

(13 + 5 5 )(13 – 5 5 )
B A
C (5,7)
GM of p&q = pq =
T(2,0)

132 – (5 5 )
2
=

= 169 – 125 B

52
= 44
= 2 11
EXAMPLES
7. If (1+ α x)n = 1+8x+24x2+................. and a line through P( α ,n)
cuts the circle x2+y2 = 4 in A and B, then PA.PB is
(a) 4 (b) 8 (c) 16 (d) 32.
Solution :
(1+ α x)n = 1+8x+24x2+.................
1+n α x+ n C 2 (αx ) +.................= 1+8x+24x2+....................
2

n α x = 8x n C 2 (αx ) 2 = 24x2 P(2,4)

n ( n – 1) 2 A
nα = 8 α = 24
2
nα ( nα – α ) T(2,0)
= 24
2
(8 – α ) B
=3
2
⇒ 8– α = 6
α =2
∴ n = 4.
P( α ,n) = P(2,4) and PT is a tangent of length 4
We know that
PT2 = PA.PB = 42 = 16
(Secant tangent theorem)
8. If ƒ(x+y) = ƒ(x)ƒ(y) ∀ x, y, ƒ(1) = 2 and α n = f(n), n ∈ N then the equation of the circle having
( α 1, α 2) and ( α 3, α 4) as the ends of its one diameter is
(a) (x–2)(x–8)+(y–4)(y–16) = 0 (b) (x–4)(x–8)+(y–2)(y–16) = 0
(c) (x–2)(x–16)+(y–4)(y–8) = 0 (d) (x–6)(x–8)+(y–5)(y–6) = 0
Solution :
ƒ(x+y)=ƒ(x)ƒ(y)
ƒ(2)= ƒ(1+1) = ƒ(1)ƒ(1) = 22 = α 2
α 3=ƒ(3)=23, ƒ(4) = 24= α 4
( α 1, α 2)= (2,4) & ( α 3, α 4) = (8, 16)
Equation of circle in diameter form
(x–x1) (x–x2) + (y–y1)(y–y2) = 0
(x–2) (x–8) + (y–4)(y–16) = 0

53
9. If A and B are two points on the circle x2+y2–4x+6y–3 = 0 which are farthest and nearest respec-
tively from the point (7, 2) then
(a) A ≡ (2–2 2 , –3–2 2 ) (b) A ≡ (2+2 2 , –3+2 2 )

(c) β ≡ (2+2 2 , –3+2 2 ) (d) β ≡ (2–2 2 , –3+2 2 )


Solution :
x2+y2–4x+6y–3 = 0
Centre of the circle is (2, –3)

Radius of the circle is = 22 + 33 + 3 = 16 = 4


S1 = 49+ 4–28+12–3
A P(7,2)
= 65–31 C (2,-3) B

= 34>0
∴ point (7, 2) lies outside the circle

PC = (7 – 2)2 + (2 + 3)2
= 52 + 52
= 50 = 5 2
CA = CB = r = 4
Farthest point PA = PC+CA
=5 2 +4
Nearest point PB = PC–CB
= 5 2 –4
By tinding Point of pnteraction
∴ The coordinates of A and B are
(( )( )) (
A 2 – 2 2 , – 3 – 2 2 and B 2 + 2 2 ,–3 + 2 2 )
PRACTICE QUESTIONS
PA
1. The points A and B in a plane are such that for all points P lies on circle Satisfying = k, then k
PB
will not be equal to
(a) 0 (b) 1 (c) 2 (d) None of these
2 2 2
2. If the line hx+ky = 1 touches x +y = a then the locus of the point (h,k) is a circle of radius
1
(a) a (b) 1/a (c) a (d)
a
3. Equation of incircle of equilateral triangle ABC where B(2,0), C(4,0) and A lies in the fourth
54
quadrant is
2y 2y
(a) x2+y2–6x+ +9 = 0 (b) x2+y2–6x– +9 = 0
3 3
2y
(c) x2+y2+6x+ +9 = 0 (d) None of these
3
4. A variable circle having chord of radius ‘a’ passes through origin meets the coordinate axes in
points A and B. locus of centroid of triangle OAB, ‘O’ being the origin is
(a) 9(x2+y2) = 4a2 (b) 9(x2+y2) = a2
(c) 9(x2+y2) = 2a2 (d) 9(x2+y2) = 8a2
5 The locus of the centre of the circle which cuts a chord of length 2a from the positive x–axis and
passes through a point on positive y–axis distant b from the origin is
(a) x2+2by = b2+a2 (b) x2–2by = b2+a2
(c) x2+2by = a2–b2 (d) x2–2by = b2–a2
6 The number of circle having radius 5 and passing through the points (–2,0) and (4,0) is
(a) one (b) two (c) four (d) infinite
7 The equation of the smallest circle passing through the intersection of the line x+y=1 and the circle
x2+y2 = 9 is
(a) x2+y2+x+y–8 = 0 (b) x2+y2–x–y–8 = 0
(c) x2+y2–x–y+8 = 0 (d) None of these
8 The number of the points on the circle x2+y2–4x–10y+13=0 which are at a distance 1 from the
point (–3,2) is
(a) 1 (b) 2 (c) 3 (d) None of these
9 The locus of the mid-point of a chord of the circle x2+y2 = 4 which subtends a right angle at the
origin is
(a) x+y=2 (b) x2+y2=1 (c) x2+y2=2 (d) x+y = 1
10 The area of the triangle formed by joining the origin to the points of intersection of the line x 5 +2y
=3 5 and circle x2+y2 = 10 is
(a) 3 (b) 4 (c) 5 (d) 6
11 If (–3,2) lies on the circle x2+y2+2gx+2fy+c=0 which is concentric with the circle x2+y2+6x+8y–
5 = 0 then c is
(a) 11 (b) –11 (c) 24 (d) None of these
12 A variable circle passes through the fixed point A(p,q) and touches x–axis. The locus of the other
end of the diameter through A is
(a) (x–p)2=4qy (b) (x–q)2=4qy (c) (y–p)2=4qx (d) (y–q)2=4px
13 If the lines 2x+3y+1 = 0 and 3x–y–4 = 0 lie along diameters of a circle of circmference 10 π , then

55
the equation of the circle is
(a) x2+y2–2x+2y–23=0 (b) x2+y2+2x+2y–23=0
(c) x2+y2–2x–2y–23=0 (d) x2+y2+2x–2y–23=0
14 The lines 2x–3y=5 and 3x–4y = 7 are diameters of a circle having area as 154 sq.unit, then the
equation of the circle is
(a) x2+y2+2x–2y=62 (b) x2+y2+2x–2y=47
(c) x2+y2–2x+2y=47 (d) x2+y2–2x+2y=62
15 The equation of circle which passes through the origin and cuts off intercepts 5 and 6 from the
positive parts of the axes respectively, is
61 61
(a) (x+5/2)2+(y+3)2= (b) (x–5/2)2+(y–3)2=
4 4
61 61
(c) (x–5/2)2–(y–3)2= (d) (x–5/2)2+(y+3)2=
4 4

ANSWERS
1. b 2. b 3.a 4. a 5.c 6. b
7.b 8. d 9. c 10. c 11. b 12. a
13. a 14. c 15. b

56
CIRCLE - IV
Tangents and Normals
Topics covered :
1. Distance of a line from a circle
2. Different forms of the equations of tangents
3. Length of tangent from a point to a circle.
4. Normal to a circle at a given point.

1. Line and a circle


Let S = 0 and L = 0 be a circle and a line. If r is the radius of the circle and d is the length of
perpendicular from the centre on the line then,

S=0
C

(i) d>r ⇒ line does not meet circle d

L=0

S=0
C
(ii) d=r ⇒ line touches the circle. It is a tangent to the circle d r

L=0

S=0
C
(iii) d<r ⇒ line intersect the circle line is a secant to the circle d r

L=0

S=0

(iv) d=0 ⇒ line is a diameter of the circle L=0

If y = mx+c is a line and x2+y2 =r2 is a circle then


(i) c2 > r2 (1+m2) ⇒ The line is a secant of the circle.The line intersects the circle in two
distinct points.
(ii) c2 = r2 (1+m2) ⇒ The line is a tangent to the circle.The line touches the circle at unique
point.
(iii) c2 < r2 (1+m2) ⇒ The line does not meet the circle

57
Equations of Tangents
1 Point form : Equation of the tangent to the circle x2+y2 = a2 at the point (x1, y1) on it is :
xx1+yy1=a2
2 Equation of the tangent to the circle x2+y2+2gx+2fy+c=0 at the point (x1, y1) on it is
xx1+yy1+g(x+x1)+ƒ(y+y1)+c = 0
I. Parametric forms
Equation of the tangent to the circle x2+y2 = a2 at the point (a cos θ ,asin θ ) on it its xcos θ +ysin θ
=a
II. Slope form
The equation of a tangent of slope m to the circle x2+y2 = a2 is y = mx ± a 1+ m 2

⎛ am a ⎞
The coordinates of the point of contact are ⎜⎜ ± ,+ ⎟⎟
⎝ 1+ m 1 + m2 ⎠
2

(i) Condition for a line y = mx+c to be a tangent to the circle x2+y2=a2 is c2 =a2(1+m2) or c =
± a 1+ m 2
(ii) Condition that the line x+my+n = 0 touches the circle x2+y2+2gx+2fy+c = 0 is ( g+mf+n)2
= ( 2+m2)(g2+ƒ2–c)
(iii) Equation of tangent to the circle x2+y2+2gx+2fy+c = 0 in terms of slope is y = mx+mg–
f± g2 + ƒ2 – c 1+ m 2
(iv) The line x+my+n = 0 touches the circle (x–a)2+(y–b)2 = r2 if (a +bm+n)2 = r2( 2+m2)
(v) If the line y = mx+c is the tangent to the circle x2+y2 = r2 then point of contact is given by
⎛ – mr 2 r 2 ⎞
⎜⎜ , ⎟⎟
⎝ c c⎠
(vi) If the line ax+by+c = 0 is the tangent to the circle x2+y2 = r2 then point of contact is given
⎛ – ar 2 – br 2 ⎞
by ⎜⎜ c , c ⎟⎟
⎝ ⎠
III. Tangents from a point outside the circle
If circle is x2+y2 = a2 and
any tangent to the circle is
A
y = mx+a 1+ m 2
a
P(x1, y1) lies on the tangent
∴ y1 = mx1+a 1+ m 2 C (0,0) P(x1,y1)

y1–mx1 = a 1+ m 2
B
Squaring
(y1–mx1)2 = a2(1+m2)
m2(x12–a2) – 2mx1y1+y12–a2 = 0 is a quadratic equation
in m which gives 2 values of m.
We get two equations of tangents.

58
Tangents from a point outside the circle
Let the circle be x2+y2=r2 and a point P(x1,y1) outside
the circle .
Let the slope of the tangent is m then equation of the
tangent is y–y1 = m (x–x1)___________(1)
Now find the distance of this line from the centre (0,0)
and equate to the radius. We get equation in m solve and
get two values of m and substitute in (1)
IV Length of the tangent from a point to a circle.
Let the circle be S ≡ x2+y2+2gx+2ƒy+c = 0 then
centre and radius of circle are (–g, –ƒ) and T

g 2 + ƒ2 – c respectively and let P(x1, y1) be any


g2 + f2 − c
point outside the circle.
C (-g,-f) P(x,y)
1 1

x + y + 2 gx1 + 2 f1 + y1 + c
2 2
PT = 1 1

= S1
V. Power of point with respect to a circle
The power of P(x1, y1) with respect to S ≡
T
x2+y2+2gx+2ƒy+c = 0 is equal to PA. PB or
B
PC.PD which is x12+y12+2gx1+2ƒy1+c = 0 ⇒
A
S1=0 (: PA.PB = PC.PD= PT2)
P(x1,y1)
Note :
C
(i) The power of the point outside the circle is posi-
tive D

(ii) The power of the point on the circle is zero


(iii) The power of the point inside the circle is negative
A
VI. Pair of tangents S=0
The equation of the pair of tangents drawn from the point
P(x1,y1) to the circle S = 0 is SS1 = T2
P(x ,y )
Where S = x2+y2+2gx+2ƒy+c 1 1

S1 = x12+y12+2gx1+2ƒy1+c
T = xx1+yy1+g(x+x1)+ƒ(y+y1)+c B
Note : The pair of tangents from (0,0) to the circle
x2+y2+2gx+2ƒy+c = 0 are at right angles if g2+f2 = 2c
VII. Normal to a circle at a given point
The normal of a circle at any point is a straight line which is perpendicular to the tangent at the
point and always passes through the centre of the circle.
(1) Point form
To find the equation of normal to the circle x2+y2 = a2 at the point p(x1,y1) on it .
Since we know that normal passes through the centre of a circle . So we get two points on normal
using two point form of a line we get the equation of normal as

59
y–0 x–0
= P(x1,y1)
y1 – 0 x1 – 0 a
or
T
C (0,0)
y x
=
y1 x 1
xy1–yx1 = 0
To find normal at (x1, y1) of second degree conics ax2+2hxy+by2+2gx+2ƒy+c = 0__________(1)
a h g
h b f
then according to determinant
g f e

Write first two rows as ax1+hy1+g and hx1+by1+f


x – x1 y – y1
Then normal at (x1,y1) of (1) is ax + hy + g = hx + by + f
1 1 1 1

 If equation of circle is x2+y2 = a2


here a= b = 1 and h = 0 = g = f
x – x1 y – y1
∴ x1 = y1

x y
–1 = –1
x1 y1

x y
x1 y1 is equation of normal at (x1, y1)
=

 If equation of circle is x2+y2+2gx+2fy+c = 0


here a = b = 1 and h = 0
x – x1 y – y1
Then x + g = y + f
1 1

60
CIRCLE - V
Tangents and Normals

1. Chord of contact
Let the equation of circle be x2+y2 = r2 PA and A
PB are pair of tangents drawn from the point
P(x1,y1) then AB is the chord of contact with A
and B as its points of contact. P(x1,y1)
∴ Equation of chord of contact AB is xx1+yy1=a2
Equation of chord of contact look like equation
of tangent at point but point are different B

If the equation of circle be x2+y2+2gx+2fy+c =


0 then the equation of chord of contact is
xx1+yy1+g(x+x1)+f(y+y1)+c=0
2. Equation of the chord bisected at a given point
Let the equation of circle be x2+y2=r2 and AB is a chord of it Let (M(x1,y1) be midpoint of AB.
y1
Slope of CM = x
1 A
M(x
1, y )
– x1 1

Slope of AB = y B
1
C (0,0)
Equation of chord AB is
– x1
y–y1= y (x–x1)
1

yy1–y12 = – xx1+x12
xx1+yy1=x12+y12
xx1+yy1–a2=x12+y12–a2
T = S1
If the equation of circle be x2+y2+2gx+2fy+c = 0 then the equation of chord which is bisected at
(x1, y1) is
`xx1+yy1+g(x+x1)+f(y+y1)+c = x12+y12+2gx1+2fy1+c

EXAMPLES :
1 Find the equation of tangent to the circle x2+y2–2ax = 0 at the point (a(1+cos α ), a sin α )
Solution :
Equation of tangent of x2+y2–2ax = 0at (a(1+cos α ),asin α ) is
ax(1+cos α )+aysin α –a(x+a(1+cos α )) = 0
ax+axcos α +aysin α –ax–a2(1+cos α ) = 0
axcos α +aysin α =a2(1+cos α )
xcos α +ysin α = a(1+cos α )
2 Find the equations of the tangents to the circle x2+y2 = 9 which make an angle of 60° with the
axis.

61
Solution :
Since tangents make an angle of 60° with the x–axis so slope of tangent
m = tan60°=m= 3
radius of circle x2+y2 = 9 is 3
we know equation of tangent to a circle x2+y2 = a2 is
y = mx ± a 1+ m 2
∴ y= 3 x ± 3 1+ 3
= 3 x± 6
or 3 x–y ± 6 = 0 ie., 3 x–y+6 = 0 and 3 x–y–6 =0 are equations of tangents.
3 Show that the line (x–2) cos θ +(y–2) sin θ = 1 touches a circle for all values of θ . Find the circle.
Solution :
Since the line (x–2) cos θ +(y–2) sin θ = 1_______(1) touches a circle so it is a tangent equation
to a circle.
Equation of tangent to a circle at (x1,y1) is (x–h)x1+(y–k)y1 = a2 to a circle (x–h)2+(y–k)2= a2
comparing (1) and (2) we get
x–h = x–2 y–k = y–2 and a2 = 1
x1 = 1cos θ y1 = 1sin θ
∴ Required equation of circle is
(x–2)2 + (y–2)2 = 1
x2+y2–4x–4y+7 = 0
4 Find the equation of the normal to the circle x2+y2 =2x which is parallel to the line x+2y = 3
Solution :
Equation of normal at (x1,y1) of x2+y2 –2x = 0 is
x – x1 y – y1 ⎛ x – x1 y – y1 ⎞
= ⎜⎜ = ⎟⎟
x1 – 1 y1 – 0 ⎝ ax1 + g by1 + f ⎠
y1
Slope of this equation is x – 1
1

–1
Slope of x+2y = 3 is
2
Since given that normal is parallel to x+2y =3
y1 –1
∴ x –1 =
1 2
2y1=–x1+1 therefore locus of (x1, y1) is x1+2y1 = 1
It is the equation of normal ⇒ (x+2y)=1
5 Show that the line 3x–4y = 1 touches the circle x2+y2–2x+4y+1 = 0 find the coordinates of the
point of contact.
Solution :
Centre and radius of circle x2+y2–2x+4y+1 = 0

62
is (1,–2) and 1+ 4 – 1 = 2 respectively.
If the distance of a line 3x–4y = 1 from the centre (1,–2) is equal to radius then the line touches or
it is tangent to a circle.

3 × 1 – 4(–2) – 1 ⎛ ax1 + by1 + c ⎞


⎜ = d⎟
=
32 + 4 2 ⎜ a 2 + b2 ⎟
⎝ ⎠
3+ 8 –1
=
5
=2
∴ line 3x–4y= 1 touches the circle.
Let point of contact be (x1,y1) then equation of tangent to a circle x2+y2–2x+4y+1=0 is
xx1+yy1–(x+x1)+2(y+y1)+1 = 0_________________(1)
x(x1–1)+y(y1+2)–x1+2y1+1 = 0__________________(2)
and given line 3x–4y–1 = 0
(1) and (2) are idenfical then comparing (1) and (2) we get
x1 – 1 y1 + 2 – x1 + 2 y1 + 1
= =
3 –4 –1
–x1+1= –3x1+6y1+3 or 2x1–6y1–2=0
–y1–2 =4x1–8y1–4 or 4x1–7y1–2=0
–1 –2
Solving these two equations of x1,y1 we get x1 = and y1 =
5 5
⎛ –1 – 2 ⎞
∴ point of contact is ⎜ 5 , 5 ⎟
⎝ ⎠
6 The angle between a pair of tangents from a point P to the circle
x2+y2+4x–6y+9sin2 α +13cos2 α = 0 is 2 α . Find the equation of the locus of the point P.
Solution :
Let the coordinate of Pbe (x1,y1) and given circle is x2+y2+4x–6y+9sin2 α +13cos2 α =0
(x+2)2+(y–3)2–4–9+9sin2 α +13cos2 α = 0
(x+2)2+(y–3)2–9sin2 α –13(1–cos2 α ) = 0
(x+2)2+(y–3)2–9sin2 α –13sin2 α = 0
(x+2)2+(y–3)2 = 4sin2 α = (2sin α )2
∴ centre and radius of circle is (–2,3) and 2 sin α respectively
Distance of P and C is
T
PC = ( x 1 + 2) 2 + ( y1 – 3) 2
2sinα
In Δ PCR
C α
2 sin α (-2,3) α 2α P(x1,y1)

sin α = ( x 1 + 2) 2 + ( y1 – 3) 2
R

63
or ( x 1 + 2) 2 + ( y1 – 3) 2 = 2
Squaring
(x1+2)2+(y1–3)2 = 4 or (x+2)2+(y–3)2 = 4
∴ locus of point P is a circle
7 If the length of tangent from (f,g) to the circle x2+y2 =6 be twice the length of the tangent from (f,g)
to circle x2+y2+3x+3y = 0 then prove that f2+g2+4f+4g+2 = 0
Solution :
According to the question
f 2 + g 2 – 6 = 2 f 2 + g 2 + 3f + 3g
Squaring both side
f2+g2–6 = 4 (f2+g2+3f+3g)
3f2+3g2+12f+12g+6 = 0
Divide by 3 we get f2+g2+4f+4g+2 = 0

8 The chord of contact of tangents drawn from a point on the circle x2+y2 = a2 to the circle x2+y2 =
b2 touches the circle x2+y2 = c2 show that a,b,c are in GP.
Solution :
Let P (a cos θ ,asin θ ) be a point on the circle x2+y2 = a2 Then equation of chord of contact to the
circle x2+y2 = b2 from P (a cos θ , asin θ ) is
x(a cos θ )+y (asin θ ) = b2
T
axsin θ +aysin θ = b2 P

It is a tangent to the circle x2+y2= c2


o
∴ length of perpendicular to the line = radius.
R 2 2
x +y =c
2

2
–b 2 2
x +y =b
2

2 =c x2+y2=a2
a
2
b = ac
∴ a, b.c are in G.P.

64
CIRCLE - VI
Tangents and Normals

1 Director circle and its equation


The locus of the point of intersection of two perpendicular tangents to a given circle is known as
its director circle.
Equation of Director Circle
P(h,k)
90°
45° 45°

A B
r r
C(0, 0)

Let P(h,k) be the point of intersection of tangents to a circle x2+y2 = r2at right angle.
ACBP is a square
∴ AC = CPsin45°
h2 + k2
r=
2
Squaring we get
2r2 = h2+k2
or x2+y2 = 2r2 is the required equation. of director circle
2 Intersection of two circles, common Tangents to two circles
Let the two circles be (x–g1)2+(y–f1)2 = r12 and (x–g2)2+(y–f2)2 = r22
with centres C1(g1,f1) and C2(g2,f2) and radii r1 and r2 respectively
Different cases of intersection of two circles
Case I When |C1C2|>r1+r2
ie., distance between the centre is greater than sum of the radii.
In this case there are 4 common tangents can be drawn.
Two direct common tangents (circles lies on the same side of the tangent)
ABE and CDE

A
r1 I B
H r
2
E
C1 F C2
J D
G

Two indirect (Transverse) common tangents (circles lies opposite side of the tangent)
GFH & IFJ.
Note that centres of two circles and point of intersection of tangents are collinear also

65
C1E r1 C1F r1
C2E = r2 & C2 F = r2
To find the equations of common tangents.
Let us assume equation of tangent of any circle in slope form be
(y+f) = m(x+g)+r 1+ m 2
Points E & F satisfy this equation. Substitute the coordinates of E & F to get the values of m.
Substitute the values of m we get required respective common tangent equation
Case II When|C1C2| = r1+r2

A H
B

E
C1 C2

D
C
G
ie., distance between the centre is equal to sum of the radii.
In this case there are 3 common tangents 2 direct common tangent and one transverse common
tangent
The equation of tangent at point F is S1–S2 = 0 where S1 = 0 and S2 = 0 are equations of circles.
⎛ r1g 2 + r2g1 r1f 2 + r2f1 ⎞
Coordinate of F are ⎜⎜ r r , r r ⎟⎟
⎝ 1+ 2 1+ 2 ⎠
Case III When |C1C2|<r1+r2
A
B

E
C1 C2

D
C

ie,. Distance between the centre is less than sum of the radii.
In this case only two direct common tangents.
Case IV When |C1C2| = |r1–r2|

S1
S2
F
C1 C2

66
ie., distance between the centre is equal to difference of the radii.
Then the two circles touch each other internally.
In this case only one direct common tangent.
Equation of common tangent is S1–S2 = 0
F divides line joining C1 and C2 externally in the ratio r1:r2
⎛ r1g 2 – r2 g1 r1f 2 – r2 f1 ⎞
∴ coordinates of F are ⎜⎜ , ⎟⎟
⎝ 1 2
r – r r 1 – r2 ⎠
Case V When |C1C2|<|r1–r2|

S1
S2

C1 C2

ie., distance between the centre is less than the difference of the radii. Then one circle contains the
other
In this case there is no real Common tangents.

67
3 Length of direct common tangent if |C1C2|>r1+r2
length of direct common tangent = d 2 – ( r1 – r2 ) 2

length of transverse common tangent = d 2 – ( r1 + r2 ) 2


Where d = |C1C2| and r1, r2 are radii of the circles
4 Pole and Polar of the circle
Let P(x1,y1) be any point inside or outside the circle. Draw chords AB and CD passing through P
. If tangents to the circle at A and B meet at Q (h,k) then locus of Q is called the polar of P with
respect to circle and P is called the pole and if tangents to the circle at
C and D meet at R, then the straight line QR is polar with P as its pole.
Let the equation of circle be x2+y2 = a2 then AB is a chord of contact of Q(h,k)
∴ xh+yk = a2 is its equation
P (x1,y1) lies on AB
∴ hx1+ky1 = a2.

R D

A
Polar
P(x1,y1)pole

Q B
(h,k)

Hence locus of Q(h,k) is xx1+yy1=a2 which is polar of P(x1,y1) with respect to circle x2+y2= a2
Equation of polar of the circle x2+y2+2gx+2fy+c = 0 with respect to (x1,y1) is
xx1+yy1+g(x+x1)+f(y+y1)+c = 0
Coordinates of pole of a line
Let the polar line ax+by +c = 0 with respect to the circle x2+y2 = r2
Let the pole be (x1,y1) then equation of polar with respect to the circle x2+y2 = r2 is
xx1+yy1 – r2 = 0____________________(1)
Which is same as ax+by+c =0______________(2)

comparing (1) and (2) we get


x1 y1 – r 2
= =
a b c
– ar 2 – br 2
x1 = & y1 =
c c

68
⎛ – ar 2 – br 2 ⎞
∴ The coordinate of pole is ⎜⎜ c , c ⎟⎟
⎝ ⎠
Properties of pole and polar
(i) The distance of any two points P(x1,y1) and Q(x2, y2) from the centre of a circle is proportional to
the distance of each from the polar to the other
(ii) If O be the centre of a circle and P any point then OP is perpendicular to the polar of P
(iii) If O be the centre of a circle and P any point then if OP (produced if necessary) meet the polar
of P in S then OP.OS = r2
(iv) If the polar of P(x1,y1) with respect to a circle passes through S(x2,y2) then the ploar of S will pass
through P and such points are said to be conjugate points
(v) If the pole of the line ax+by+c=0 with respect to a circle lies on another line a1x+b1y+c1 = 0.
Then the pole of the second line will lie on the first line and such lines are said to be conjugate
lines.
Note :
(i) P(x1,y1) and Q(x2,y2) are convent points w.r.t the circle x2+y2+2gx+2fy+c = 0 if
x1x2+y1y2+g(x1+x2)+f(y1+y2)+c = 0
(ii) If P and Q are confined points w.r.t to a circle with centre at O and radius r, then
PQ2 = OP2+OQ2–2r2.
5 Common Chord of two circles
The common chord joining the point of intersection of two circles is called their common chord .If
S = 0 and S1 = 0 be two interesting circles then the equation of their common chord is S–S1 = 0
Let the equations of circle one
S = x2+y2+2g1x+2f1y+c1 = 0
S1 = x2+y2+2g2x+2f2y+c2 = 0
Then equation of common chord AB is
S–S1 = 0

C1 mC2

S′ = 0
S=0
2x(g1–g2)+2y(f1–f2)+c1 –c2 = 0
Length of common chord is 2AM = (C 1 A) 2 – (C 1 M) 2
C1M is the length of perpendicular from the centre C1 to common chord and C1A is radius of
circle.
Note :
(i) Common chord AB becomes maximum length when it is a diameter of the smaller one.
(ii) Circle on the common chord a diameter then centre of the circle passing through A and B lie on
the common chord of the two circle
(iii) If the length of common chord is zero then the two circles touch each other and the common

69
chord be comes the common tangent to the two circles at the point of contact.
Examples
1 There are two circles whose equations are x2+y2 = 9 and x2+y2–8x–6y+n2 = O n ∈ Z. If two
circles have exactly two common tangents then the number of possible values of n
Solution : For x2+y2 = 9 centre is (0,0) and radius 3 and for x2+y2–8x–6y+n2 = 0 centre is (4,3)
and radius 4 2 + 32 + n 2 = 25 – n 2
We know that to get exactly two common tangents the circles must intersect is|C1C2|<r1+r2
4 2 + 32 < 3+ 25 – n 2
5< 3+ 25 – n 2

2< 25 – n 2
4< 25–n2
n2<21
or – 21 < n< 21 and 25–n2 ≥ 0
25 ≥ n2
or –5 ≤ n ≤ 5
n∈ Z. so n = –4, –3, –2, –1, 0, 1, 2, 3, 4
∴ number of possible value of n is 9
2 Find all the common tangents to the circles x2+y2–2x–6y+9 = 0 and x2+y2+6x–2y+1 = 0
Solution : Centre and radius of circle x2+y2–2x–6y+9 = 0 is C1(1,3) and r1 = 1 respectively
Centre and radius of circle x2+y2+6x–2y+1 = 0 is C2(–3,1) and r2 = 3
C 2C 2 = 42 + 22 r1+r2 = 1+3 = 4
= 16 + 4 ∴ |C1C2|>r1+r2
= 20 ∴ These are four common tangents
=2 5 2 direct and 2 transverse.
To find equations of direct common tangents
Since the coordinate of E divides the line C1C2 in the ratio r1:r2 ie., 1:3 externally
∴ coordinate of E is
⎛ 1(–3) – 3.1 1.1 – 3.3 ⎞
⎜ , ⎟ or E (3,4)
⎝ 1– 3 1– 3 ⎠

A
3
B
1
E
C2(–3,1) C1(1,3)
D

70
Equation of AB through E (3,4) with slope m is
y–4= m1(x–3) or m1x–y+(4–3m1) = 0__________________(1)
If is a tangent to the circle so distance of the line from the centre is equal to radius

– 3m1 – 1 + 4 – 3m1
∴ 3=
m1 + 1
2

3 – 6 m1
3= `
m1 + 1
2

3 – 6m1
3=
m1 + 1
2

m1 + 1 = 1–2m1
2

Squaring
m12+1 = 1+4m12–4m1
3m12–4m1 = 0 ⇒ m1 = 0 or m1 = 4/3
Substitute the values of m1 in equation (1) we get the equations of tangents
y = 4 and 4x–3y = 0
To find the indirect or transverse common tangents
The coordinates of T dividing
C1C2 internally in the ratio
r1: r2 = 1: 3 is
⎛ – 3 + 3 1+ 9 ⎞ ⎛ 5 ⎞
⎜ , ⎟ = ⎜ 0, ⎟
⎝ 4 4 ⎠ ⎝ 2⎠
Equation of line through T with slope m2 is
y –5/2 = m2(x–0) or 2m2x–2y+5 = 0

R
Q

C2(–3,1) T C1(1,3)
S
P

Since it is a tangent to the circles

71
∴ distance of the line from its centre is equal to radius.

2m 2 (–3) – 2(1) + 5
∴ =3
4m 2 + 4
2

– 6m 2 + 3
=3
2 m2 + 1
2

m2 + 1
2
or 3–6m2 = 6

1–2m2 = 2 m 2 2 + 1
Squaring
1+4m22–4m2= 4m22+4
3
4m2 = –3 or m2 = and ∞ (as coefficient of m22 = 0)
4
∴ The equations of transverse common tangents are
3x+4y–10 = 0 and x = 0
3 If the circle C1 = x2+y2 = 16 intersects another circles C2 of radius 5 in such a manner that the
common chord is of maximum length and has a slope equal to 3/4 the find the coordinates of the
centre C2
⎛ 9 – 12 ⎞ ⎛ – 9 12 ⎞
Solution: ⎜ , ⎟&⎜ , ⎟
⎝5 5 ⎠ ⎝ 5 5 ⎠
When two circles intersect, the common chord of maximum length will be the diameter of smaller
circle.
∴ AB is diameter of smaller circle
C2A = 5 and C1A = 4

A
5 4
3
C2(h,k) C1(0,0)

∴ C1C2 = 5 2 – 4 2
C1C2 = 3
Given that slope of AB is 3/4
∴ slope of C1C2 is –4/3

72
k
and slope of C1C2 is
h
–4 k
∴ =
3 h
–4
k= h
3
C1C2 = h2 + k2 = 3
or h2+k2 = 9__________________(1)
–4
Substitute k = h in equation (1)
3
16 2
h2+ h =9
9
25h2 = 81
81
h2 =
25
9 12
h= ± ∴K = ±
5 5
⎛ 9 – 12 ⎞ ⎛ – 9 12 ⎞
∴ coordinates of C2 are ⎜ 5 , 5 ⎟ and ⎜ 5 , 5 ⎟
⎝ ⎠ ⎝ ⎠
4 The circle x2+y2–4x–8y+16 = 0 rolls up the tangent to it at (2+ 3 ,3) by 2units, assuming the x-
axis as horizontal, find the equation of the circle in the new position.
Solution ;
Given circle is x2+y2–4x–8y+16 = 0
Equation of tangent to the circle at (2+ 3 ,3) is (2+ 3 )x+3y–2(2+ 3 +x)–4(y+3)+16 = 0
3 x–y–2 3 = 0
slope of this line tan θ = 3 ∴ θ = 600.
A and B are the centres of
The circles in old and new positions respectively then

A
(2,4) B

P Q
(2 + 3 ,3)

73
A (2,4) and B(2+2cos600), 4+2sin60) ( ⇒ AB = 2)
∴ B(3,4+ 3 )
Hence equation of circle at new position is
(x–3)2+(y–4– 3 )2 = 22
x2+y2–6x–2(4+ 3 )y+24+8 3 =0
5 The pole of a straight line with respect to the circle x2+y2=a2 lies on the circle x2+y2=9a2
a2
Prove that the straight line touches the circle x2+y2 =
9
Solution Let pole be (x1,y1)
The equation of the polar with respect to x2+y2 = a2is
xx1+yy1 = a2
But given that P(x1,y1) lies on the circle x2+y2 = 9a2
∴ x12+y12 = 9a2
If straight line xx1+yy1 = a2 touches the circle then distance of this line from the centre of the circle
x2+y2 = a2/9 must be a/3(radius)

– a2
ie.,
x 1 + y1
2 2

a2 a2 a2 a
2 2 = = = = radius
x 1 + y1 9a 2 3a 3
Hence straight line xx1+yy1 = a2 touches the circle x2+y2 = a2/9
6 Prove that the polar of a given point with respect to any one of the circles x2+y2–2kx+c2 = 0,
where k is a variable, always passes through a fixed point, whatever be the value of k.
Solution: Let P(x1,y1) be pole, then equation of polar with respect to the circle x2+y2–2kx+c2 = 0
is
xx1+yy1–k(x+x1)+c2=0
(xx1+yy1+c2)–k(x+x1) = 0___________________(1)
L1+ λ L2 = 0
Where L1 = xx1+yy1+c2 = 0 & L2 = x+x1 = 0
Hence equation (1) represents the family of lines passing through the point of intercection of two
lines L1 & L2
Solving equation L1 = 0 & L2 = 0 we get
x = –x1 & –x12+yy1+c2 = 0

x12 – c2
y=
y1

74


–x ,
( )
x 1 2 – c 2 ⎞⎟
∴ The fixed point is ⎜⎜ 1 y1 ⎟ is independent of k.

⎝ ⎠
PRACTICE QUESTIONS
1 Passage 1
Let a straight line be drawn from a point P to meet the circle in Q and R. Let the tangents at Q and
R meet at T. The locus of T is called the polar of P with respect to the circle . The given point P is
called the pole of the polar line
Let P(x1,y1) be the given point lying outside the circle in fig (i) and inside the circle in fig (ii)
Through P, draw a line to meet the circle in Q and R. Let the tangents to the circle at Q and R
meet in T (h,k).
It is required to find the polar of P ie., locus of T.
Equation of QR the chord of contact of the tangents draw from T to the circle x2+y2 = a2 is xh+yk
= a2_______________(1)
since (1) passes through P(x1,y1)
∴ x1h+y1k = a2

y T(h,k)
T y
(h,k)
R
p(x1,y1)
R
Q
p Q
o x o (x1,y1) x

∴ The locus of (h,k) is xx1+yy1 = a2, which is the equation of polar of P.


1 If the polar of P with respect to the circle x2+y2 = a2 touches the circle (x–f)2+(y–g)2 = b2 then its
locus is given by the equation.
(a) (fx+gy–a2)2 = a2(x2+y2) (b) (fx+gy–a2)2 = b2(x2+y2)
(c) (fx–gy–a2)2 = a2(x2+y2) (d) None of these.
2 The pole of the line 3x+4y = 45 with respect to the circle x2+y2–6x–8y+5 = 0 is
(a) (6,8) (b) (6,–8)
(c) (–6,8) (d) (–6,–8)
3 The pole of the chord of the circle x2+y2 = 16 which is bisected at the point (–2,3) with respect
to the circle is
⎛ – 32 48 ⎞ ⎛ 32 48 ⎞
(a) ⎜ , ⎟ (b) ⎜ , ⎟
⎝ 13 13 ⎠ ⎝ 13 13 ⎠

75
⎛ – 32 – 48 ⎞
(c) ⎜ , ⎟ (d) None of these
⎝ 13 13 ⎠
4 The coordinates of the poles of thre common chord of the circles x2+y2 = 12 and
x2+y2–5x+2y–2 = 0 with respect to the circle x2+y2 = 12 are
⎛ – 12 ⎞ ⎛ – 12 ⎞
(a) ⎜ 6, ⎟ (b) ⎜ – 6, ⎟
⎝ 5 ⎠ ⎝ 5 ⎠

⎛ 12 ⎞
(c) ⎜ 6, ⎟ (d) None of these
⎝ 5⎠

Q.5 The matching grid


I The number of common tangents to the circles
x2+y2–6x–2y+9 = 0 and x2+y2–14x–8y+61 = 0 is (a) 3
II The number of common tangents to the circles
x2+y2 = 4 and x2+y2–8x+12 = 0 is (b) 4
III The number of common tangents to the circles
x2+y2 = 4 and x2+y2–6x–8y–24 = 0 is (c) 2
IV The number of tangents to the circle
x2+y2–8x+6y+9 = 0 which pass through the point (3,–2) is (d) 1

Answer I → b(4) II → a(3)


III → d(1) IV → c(2)
Q.6 If the line x cos θ +ysin θ = 2 is the equation of a transverse common tangent to the circles x2+y2
= 4 and x2+y2–6 3 x–6y+20 = 0, Then the value of θ is
(a) 5 π /6 (b) 2 π /3 (c) π /3 (d) π /6
Q.7 Two circles of radii 4cm and 1cm touch each other externally and θ is the angle contained by their
direct common tangents. Then sin θ is equal to
24 12
(a) (b)
25 25
3
(c) (d) None of these.
4
Answer
1. b 2. a 3. a 4. a 6. d 7.a
EXAMPLES
1 If the tangent at the point P on the circle x2+y 2+6x+6y = 2 meets the straight line
5x–2y+6 = 0 at a point Q on the y–axis, then the length of PQ is
(a) 4 (b) 2 5 (c) 5 (d) 3 5
Solution C
Q (0,3) is a point where the line 5x–2y+6 = 0 meets y–axis and the trangent drawn at P
PQ is a tangent

76
y

Q(0, 3)

x
O
P

(–3,–3)

length of tangent PQ = S1 = 0 + 9 + 0 + 6×3 – 2


= 9 + 18 – 2
= 25
=5
2 If the two circles x +y +2gx+2fy = 0 and x2+y2+2g1x+2f1y = 0 and touch each other, then
2 2

(a) f1g = fg1 (b) ff1 = gg1


(c) f2+g2 =f12+g12 (d) None of these
Solution (a)
Centres of the circles are (–g,f) and (–g1,–f1)and radius of the circles are g 2 + f 2 and g12 + f12
If two circles touch each other then
|C1C2| = |r1+r2| (external)
or |r1–r2| (internal)

∴ ( g – g 1 ) 2 + ( f – f1 ) 2 = g2 + f 2 ± g1 + f1
2 2

Squaring both sides

C1 C2
r1 r2 or
(–g,–f) (–g1,–f1)

(g–g1)2+(f–f1)2 = g2+f2+g12+f12 ± 2 g 2 + f 2 g1 + f1
2 2

g2+g12–2gg1+f2+f12–2ff1 = g2+f2+g12+f12 ± 2 g 2 g12 + g 2f12 + f 2g12 + f 2f12

g 2 g1 + g 2 f1 + f 2g1 + f 2 f1
2 2 2 2
– (gg1+ff1) = ±
g2g12+f2f12+2gg1ff1=g2g12+g2f12+f2g12+f2f12
g2f12+f2g12–2gfg1f1 = 0 = (gf1–fg1)2
∴ gf1–fg1 =0 ⇒ gf1=fg1

77
3 The circles (x–a)2+(y–b)2 = c2 and (x–b)2+(y–a)2 = c2 touch each other, then
(a) a=b ± 2c (b) a = b ± 2c
(c) a = b ± c (d) None of these
Solution
Distance between centres = |r1 ± r2|
Distance between (a,b) and (b,a) = |2c| or 0
(a – b) 2 + ( b – a ) 2 = ± 2c

2 (a–b) = ± 2c
a–b = ± 2 c
a=b ± 2c

78
CIRCLE - VII
Intersection of Two Circles

Angle of intersection of two circles


Let the two circles S ≡ x2+y2+2gx+2fy+c = 0 and S′ ≡ x2+y2+2g1x+2f1y+c1=0 intersect
each other at the point P and Q. The angle θ between two circles S = 0 and S′ = 0 is
defined as the angle between the tangents to the two circles at the point of intersection.
θ must be taken acute angle .
C1 and C2 are the centres of circles S = 0 and S′ = 0 then C1(–g,–f) and C2(–g1,–f1) and their
radii

g 1 + f1 – c 1
2 2
r1 = g 2 + f 2 – c & r2=

A B
P
r1 r2
θ
C1 C2
(-g,-f) (-g1,-f1)
B' A'
Q S'=0
S=0

Let d = |c1c2| = ( g – g 1 ) 2 + (f – f 1 ) 2 = g 2 + g1 2 – 2gg 1 + f 2 + f12 – 2ff1


C1P ⊥A A ′ , C2P⊥B B′ since radius is perpendicular to the tangent at the point of contact
ie. ∠ C1P A ′ = 900 and ∠ C2P B′ = 900
∴ ∠ C1P B′ = 900 - θ and ∠ C2P A ′ = 900– θ
Hence ∠ C1PC2 = 900– θ + θ +900– θ = 1800– θ
Now in Δ C1PC2

r1 2 + r2 2 – d 2
cos(180 – θ ) =
0
(cosine rule)
2r1r2

r12 + r2 2 – d 2
cos θ = 2r1r2
If the angle between the circles is 900 ie., θ = 900, then cos900 = 0 Then the circles are said to be
orthogonal circles or the circles cut each other orthogonally.
∴ r12+r22–d2 = 0
g2+f2–c+g12+f12–c1–g2–g12+2gg1–f2–f12+2ff1= 0
2gg1+2ff1= c+c1
It is a condition for two circles to be orthogonal
Examples :
1 Find the angle between the circles

79
S ≡ x2+y2–4x+6y+11= 0 and S′ = x2+y2 –2x+8y+13 =0

Solution
Here S ≡ x2+y2–4x+6y+11= 0 and S′ = x2+y2 –2x+8y+13 =0
The centre of these circles are C1(2,–3) and C2(1,–4) respectively.
The radius of these circles are r1 = 4 + 9 – 11 = 2
and r2 = 1 + 16 – 13 = 2
Distance between the centres C1C2 = d = 12 + 12 = 2

r12 + r2 2 – (c1c 2 ) 2 2+4–2 1 π


Angle between two circle is cos θ = = = =cos
2r1r2 2×2× 2 2 4

π
∴θ =
4
2 Find the equations of the two circles which intersect the circles x2+y2–6y+1 = 0 and x2+y2–
4y+1 = 0 orthogonally and touch the line 3x+4y+5 = 0
Solution
Let the required equation of circle be x2+y2+2gx+2fy+c = 0
This circle intersect orthogonally with circles x2+y2–6y+1 = 0 and x2+y2–4y+1 = 0
Condition for orthogonality is 2gg1+2ff1 = c+c1
∴ 0 + 2f(–3) = C+1 and 0+2f(–2) = c+1
–6f = c+1 –4f=c+1
∴ –6f = –4f ⇒ f= 0 and c = –1
Equation of circle is x2+y2+2gx–1 = 0
Centre is (–g,0) and radius g2 +1
Since the line 3x+4y+5 = 0 touches the circle

∴ distance of this line from the centre must be equal to radius g2 +1

– 3g + 5
∴ = g2 +1
9 + 16

5–3g = 5 g 2 + 1
squaring
25+9g2–30g=25g2+25
16g2+30g = 0
2g(8g+15) =0
– 15
g = 0 or g =
8
Hence equations of circles are

80
15
x2+y2 –1 = 0 and x2+y2 – x–1 = 0
4
x2+y2–1 = 0 and 4x2+4y2–15x–4 = 0
3 Prove that the two circles, which pass through (0,a) and (0,–a) and touch the line y = mx+c, will
cut or thogonally if c2= a2(2+m2)
Solution
Let the equation of the circles be
x2+y2+2gx+2fy+d = 0
This circle passes through the points (0,a) and (0,–a)
∴ a2+2fa+d= 0 __________(1) and a2–2fa+d = 0 _________(2)
(1) (2)
4fa = 0
∴ f= 0 and d = –a2
∴ The equation of circle is x2+y2+2gx–a2 = 0
Centre of this circle is (–g,0) and radies g2 + a2
Since line y = mx+c touches the circle

– mg + c
∴ 2
= g2 + a 2
m +1

c–mg = g2 + a2 m2 + 1
Squaring
c2+m2g2–2mcg = g2m2+g2+a2m2+a2
g2+2mcg+a2(1+m2)–c2 = 0
It is a quadratic in g
∴ product of the roots g1g2 = a2(1+m2)–c2
Sum of roots g1+g2 = –2mc
Now the equations of the two circles represented are x2+y2+2g1x–a2=0 and x2+y2+2g2x–a2=
0
These two circles will be orthogonal if
2g1g2 = – a2–a2
g1g2= –a2
But g1g2 = –c2+a2(1+m2)
∴ –c2+a2(1+m2) = – a2
or c2 = a2 (2+m2)
Which is the required condition

4 If the angle of intersection of the circles x2+y2+x+y = 0 and x2+y2+x–y = 0 is θ , then


equation of the line passing through (1,2) and making an angle θ with the y–axis is
Solution
⎛ – 1 – 1⎞ ⎛ –1 1⎞
Let C1 and C2 be the centres of given circles C1 ⎜ , ⎟ and C ⎜ , ⎟
⎝ 2 2 ⎠ 2
⎝ 2 2⎠

81
1 1 1 1
Also radius these two circles are r1 = + = =
4 4 2 2

1 1 1
and r2 = + =
4 4 2

r1 2 + r2 2 – d 2
cos θ =
2r1 r2

1 1
+ –1
2 2
= 1 1
2 ×
2 2
=0
π
∴θ =
2
∴ Required line is parallel to x–axis and it passes through (1, 2)
∴ Equation of line is y = 2.

5 The equation of a circle is x2+y2 = 4. Find the centre of the smallest circle touching the circle and
the line x+y = 5 2
y

45 0 135
0

(-2,0) (2,0) x+y= 5 2

Solution
Here OA = 2 radius of circle x2+y2 = 4 with centre (0,0)
The distance of (0,0) from x+y = 5 2 is

–5 2
=5
2

5–2 3
∴ The radius of the smallest circle = =
2 2
3 7
and OC = 2+ =
2 2
The slope of OA = 1 = tan θ

82
1 1
∴ cos θ = and sin θ =
2 2
⎛ 7 7 ⎞
∴ Centre (0+OC cos θ , O+OC sin θ ) = ⎜⎜ , ⎟⎟
⎝2 2 2 2⎠

83
84
CIRCLE - VIII
Radical Axis
Radical axis
The radical axis of two circles is the locus of a point which moves such that the lengths of the
tangents drawn from it to the two circles are equal.

P
P
P

T2 T1 T2
T1 T2 T1
A
T
C1 C2 C1 C2
C1 C2
B

Radical axis Radical axis


Radical axis
PT1=PT2 PT1=PT2
PT1=PT2

Consider S = x2+y2+2gx+2fy+c = 0 and S′ = x2+y2+2g1x+2f1y+c1=0


Let P(x1, y1) be any point such that |PT1|=|PT2|
or (PT1)2 = (PT2)2
x12+y12+2gx1+2fy1+c = x12+y12+2g1x1+2f1y1+c1
2x1(g–g1)+2y1(f–f1)+c–c1 = 0 (1)
∴ locus of P(x1, y1) is a straight line and equation (1) represents its equation.
Equation of radical axis is 2x(g–g1)+2y(f–f1)+c–c1 = 0
Properties of radical axis
1 Radical axis is perpendicular to the line joining the centres of the given circles.
(Slope of radical axis) × slope of line joining centres = –1
– (g – g 1 ) f – f 1
× =–1
(f – f 1 ) g – g1
2 The radical axis bisects common tangents of two circles
P

A B
M

C P’ D

As M lies on radical axis and AB is a tangent to the circles ∴ AM = BM.


Hence radical axis bisects the common tangents
3 If two circles cut a third circle orthogonally, then the radical axis of the two circles will pass
through the centre of the third circle, or the locus of the centre of a circle cutting two given circles
orthogonally is the radical axis of the given two circles
CP = CQ
85
S′′ = 0

P Q
C

C1 C2

S=0 S′ = 0

Hence C lies on the radical axis of the circles S = 0 and S′ = 0


4 The position of the radical axis of the two circles geometrically

radical axis
radical axis direct common tangent
radical axis common chord
common tansverse tangent

Radical Centre
The radical axes of three circles, taken in pairs, meet in a point, which is called their radical
centre.
Let the three circles be S1 = 0, S2 = 0 S3 = 0
OL, OM, ON be radical axes of the pair sets of circles{S1 = 0, S 2 = 0}, {S2=0, S3=0},
{S3=0, S1=0} respectively
S1=0 S2=0
L

M
N

S3=0

Equations of OL, OM and on are S1–S2 = 0 S2–S3 = 0 and S3–S1 = 0


Family of lines passes through point of intersection of lines S1–S2 = 0 and S2–S3 = 0 is
S1–S2+ λ (S2–S3) = 0
If λ = 1 then the line becomes S3–S1 = 0
86
∴ The three lines are concurrent at O
O is called radical centre .
Properties of radical centre
1 Coordinates of radical centre can be found by solving the equations S1=S2 = S3 = 0
2 The radical centre of three circles described on the sides of a triangle as diameters is the orthocenter
of the triangle.

B C
B C

3 The radical centre of three given circles will be the centre of a fourth circle which cuts all the three
circles orthogonally. and the radius of the fourth circle is the length of tangent drawn from radical
centre of the three given circles to any of these circles
(x–y)2+(y–k)2 = r2 is the fourth circle with centre (h,k) and radius r
Centre (h,k) is the radical centre of three circles and radius r is the length of tangent to one of
these three circles from radical centre (h,k)
Example:
1 The equation of the three circles are given x2+y2 = 1 ,x2+y2–8x+15 =0, x2+y2+10y+24 = 0
Determine the coordinates of the point P such that the tangents drawn from it to the circles are
equal in length
Solution : We know that the point from which lengths of tangents are equal in length is radical
centre of the given three circles
Radical axis of first two circles is
S1–S2 = 0
8x–16 = 0 or x–2 = 0___________________(1)
and radical axis of second and third is
–8x–10y–9 = 0_________________________(2)
Solving (1) and (2) equations we get
x=2, –16–10y–9 = 0
10y = –25
– 25 –5
y= =
10 2
∴ Radical centre P is (2, –5/2)
2 Find the radical centre of circles x2+y2+3x+2y+1 = 0,x2+y2–x+6y+5 = 0 and x2+y2+5x–8y+15
87
= 0. Also find the equation of the circle cutting them orthogonally.
Solution :
Let S1 ≡ x2+y2+3x+2y+1 = 0
S2 ≡ x2+y2–x+6y+5 = 0
S3 ≡ x2+y2+5x–8y+15 = 0
Equations of radical axis
S1–S2 = 0 ⇒ 4x–4y–4 = 0 or x–y–1 = 0___________(1)
S2–S3 = 0 ⇒ –6x+14y–10 = 0 or –3x+7y–5 = 0 ________(2)
Solve equations (1) & (2) we get (3, 2) as radical centre.
(1) ×3–(2)×1
– 3x + 7 y – 5 = 0
– 3x + 3y + 3 = 0
+––
4y – 8 = 0
y=2
x–2–1= 0 ⇒ x = 3
radius of fourth circle cutting these three circles orthogonally is length of tangent from this centre
to any one circle
∴r = S1

= 3 2 + 2 2 + 3.3 + 2.2 + 1
= 9 + 4 + 9 + 4 +1
= 27
=3 3
∴ Equation of circle is (x–3)2+(y–2)2= (3 3 )2
x2+y2–6x–4y–14 = 0
3 Find the radical centre of three circles described on the three sides 4x–7y+10 = 0, x+y–5= 0,
7x+4y–15 = 0 of a triangle as diameters.
Solution : We know that radical centre of 3 circles described on the three sides of a triangle as
diameters is orthocenter of the triangle.
L1 ≡ 4x–7y+10 = 0________ (1)
L2 ≡ x+y–5 = 0___________(2)
L3 ≡ 7x+4y–15 = 0__________(3)
Slope of line L1 is 4/7 and slope of line L3 = –7/4 since the product of these slopes is –1
∴ L1 and L3 are perpendicular.
We know that orthocentre of a right angled triangle is the vertex where it has right angle.
Solving (1) & (3) to get orthocentre
(1)×4+(2)×7

88
16 x – 28 y + 40 = 0
49 x + 28 y – 105 = 0
65x – 65 = 0
x=1
4–7y+10 = 0
7y+14
y=2
∴ Radical centre is (1, 2)

89
90
CIRCLE - IX
Co - Axial System of Circles

Co - Axial System of Circles


A system of circles or family of circles, every pair of which have the same radical axis are
called co-axial circles

1 The equation of family of co-axial circles when the equation of radical axis and one circle are
given

S+λL=0
S=0

S+λL=0

L = ax+by+c = 0
S ≡ x2+y2+2gx+2fy+c=0
Then equation of co-axial circle is S+ λ L = 0
2 The equation of co-axial system of a circles where the equation of any two circles of the system
are

S+λL=0

S1=0
S2=0

S1+λ(S1-S2)=0 S2+λ(S1-S2)=0

S1 = x2+y2+2gx+2fy+c = 0
S2 = x2+y2+2g1x+2f1y+c1 = 0
respectively is S1+ λ (S1–S2) = 0

and S2+ λ (S1–S2) = 0 ( λ ≠ – 1)

91
S1+ λ S2 = 0 ( λ ≠ – 1)
3 The equation of a system of co-axial circles in the simplest form is x2+y2+2gx+c = 0 where g is
variable and c is a constant
The common radical axis is the y-axis (centre on x-axis)
The equation of system of co-axial circles in the simplest form is x2+y2+2fy+c = 0 where f is a
variable and c is a constant
The common radical axis is the x–axis (centre on y axis)
y

o
x

o
x

Examples :
1 Find the equation of the system of circles co-axial with the circles x2+y2+4x+2y+1=0 and x2+y2–
2x+6y–6 = 0. Also find the equation of that particular circles whose centre lies on radical axis.
Solution :
Given circles are
S1 ≡ x2+y2+4x+2y+1=0
S2 ≡ x2+y2–2x+6y–6 =0
S1–S2 = 0
6x–4y+7 = 0
System of co-axial circle is S1+ λ (S1–S2) = 0
x2+y2+4x+2y+1+ λ (6x–4y+7) = 0
x2+y2+2x(2+3 λ )+2y(1–2 λ )+1+7 λ = 0
Centre of this circle is (–(2+3 λ ), – (1–2 λ ))
lies on radical axis
∴ 6(–2–3 λ )+4(1–2 λ )+7 = 0
–12–18 λ +4–8 λ +7 = 0
–1 –26 λ = 0
–1
λ = 26
∴ Required particular member of co-axial circle is 26(x2+y2)+98x+56y+19 = 0
2 If the circumference of the circle x2+y2+8x+8y–b = 0 is bisected by the circle

92
x2+y2–2x+4y+a = 0 then a+b equal to
(a) 50 (b) 56 (c) –56 (d) –34
Solution : (c)
Equation of radical axis (common chord of these circles) is
10x+4y–b–a = 0
Centre of first circle is (–4, –4)
Since second circle bisects the first circle
Therefore centre of first circle must lie on common chord.
∴ 10(–4)+4(–4)–b–a= 0
–40–16–(a+b) = 0
∴ a+b = –56
3 The equation of the circle passing through the point of intersection of the circles x2+y2–4x–2y = 8
and x2+y2–2x–4y = 8 and the point (–1,4) is
(a) x2+y2+4x+4y–8 = 0 (b) x2+y2–3x+4y+8 = 0
(c) x2+y2+x+y–8 = 0 (d) x2+y2–3x–3y–8 = 0
Solution : (d)
Equation of any circle passing through the point of intersection of the circles is
x2+y2–4x–2y–8+ λ (x2+y2–2x–4y–8) = 0
This circle passes through the point (–1,4
∴ 1+16+4–8–8+ λ (1+16+2–16–8) = 0
5–5 λ = 0
λ =1
Required circle is x2+y2–3x–3y–8 = 0
4 If the common chord of the circles x2+(y–b)2 = 16 and x2+y2 = 16 subtends a right angle at the
origin then b =
(a) 4 (b) 4 2 (c) –4 2 (d) 8
Solution :
The equation of common chord is
S–S1 = 0
(y–b)2–y2 = 0
b2–2by = 0
b(b–2y) = 0 b = 0 ,so
2y
∴ b = 2y or 1 =
b
The combined equation of the straight lines joining the origin to the points if intersection of
2
⎛ 2y ⎞
y = b/2 and x +y = 16 ⎜ ⎟ ⇒ b2x2+(b2–64)y2 = 0
2 2
⎝ b ⎠
This equation represents a pair of perpendicular lines
∴ b2+b2–64 = 0 ⇒ b = ± 4 2

93
94
CIRCLE - X
Limiting Points
Limiting Point
Limiting points of a system of co-axial circles are the centres of the point circles belonging to
the family (circles whose radii are zero are called point circle)
1 Limiting points of the co-axial system
Let the circle is x2+y2+2gx+c = 0
Where g is a variable and c is constant

∴ centre (–g,0) and radius g 2 – c respectively

Let g 2 – c = 0 radius
g2–c = 0
g2 = c
g= ± c
Thus we get the two limiting points of the given co-axial system as ( ) (
c, 0 & – c, 0 )
The limiting points are real and distinct, real and coincident or imaginary according
as C>,=, <0.
2 System of co-axial circles whose limiting points are given
Let ( α , β ) and ( γ , δ ) be the two given limiting points
Then corresponding circles with zero radii are
(x– α )2+(y– β )2 = 0 = x2+y2–2 α x–2 β y+ α 2+ β 2 = 0
(x– γ )2+(y– δ )2 = 0 = x2+y2–2 γ x–2 δ y+ γ 2+ δ 2 = 0
System of co-axial circle equation is
x2+y2–2 α x–2 β y+ α 2+ β 2+ λ (x2+y2–2 γ x+2 δ y+ γ 2+ δ 2) = 0 ( λ ≠ –1)
⎛ α + γλ β + δλ ⎞
centre of this circle is ⎜ , ⎟ _____________(1)
⎝ 1+ λ 1+ λ ⎠

and radius= ⎜ ⎟ +⎜
2
⎛ α + γλ ⎞ ⎛ β + δλ ⎞
⎟ –
2
(
α 2 + β 2 + λγ 2 + λδ 2 )
=0
⎝ 1+ λ ⎠ ⎝ 1+ λ ⎠ 1+ λ
After solving find λ substitute in (1)
We get the limiting point of co-axial system.
Properties of Limiting points
1 The limiting point of a system of co-axial circles are conjugate points with respect to any member
of the system.
Let the equation of any circle be
x2+y2+2gx+c = 0________________(1)
limiting points of (1) are ( )(
c, 0 , – c, 0 )

95
The polar of the point ( c, 0 is)
x c +g(x+ c )+c = 0
(x+ c ) (g+ c ) = 0
x+ c =0 ⇒ x = – c

( )
∴ – c, 0 ) lies on this.
Similarly ( ) ( )
c ,0 lies on polar with respect to – c,0
∴ These limiting points (– c ,0 ) and ( c ,0 ) are conjugate points
2 Every circle through the limiting points of a co-axial system is orthogonal to all circles of the
system
Let the equation of any circle x2+y2+2gx+c = 0 where g is a variable and c is a constant Limiting
point of this circle are – c ,0 and ( ) ( c ,0 )
(
Now let, x2+y2+2g1x+2f1y+c1 = be any circle passing through the limiting points – c ,0 and )
( )
c ,0 .
∴ c –2g1 c +c1 = 0_______(3) and c+2g1 c +c1 = 0________(4)
Solving (3) and (4). We get g1 = 0 and c1 = –c
∴ The equation becomes
x2+y2+2f1y–c = 0
applying condition of orthogonality
2gg1+2ff1=c+c1
o+o=c–c
Hence condition is satisfied for all values of g1 and f1
Examples :
1. If the origin be one limiting point of a system of co-axial circles of which x2+y2+3x+4y+25=0 is a
member, find the other limiting point.
Solution :
Equation of circle with origin (0,0) as limiting point is x2+y2=0
Given that one member of system of co-axial circle is x2+y2+3x+4y+25 = 0
∴ The system of co-axial circles is
3 4 25
x2+y2+ x+ y+ =0
1+ λ 1+ λ 1+ λ
⎛ −3 −2 ⎞
centre ⎜⎜ , ⎟⎟
⎝ 2(1 + λ ) 1 + λ ⎠
9 4 25
radius + –
4(1 + λ ) 2
(1 + λ ) 2 (1 + λ ) = 0

96
25 25
- =0
4(1 + λ )2 1+ λ
1 – 4(1+ λ ) = 0
1 + λ = 1/4
λ = 1/4 – 1 = –3/4
∴ centre (–6,–8) is the other limiting point of the system .
Example 2
Find the radical axis of co-axial system of circles whose limiting points are (–1,2) and (2,3).
Solution :
Equations of circles with limiting points (–1,2) and (2,3) are
(x+1)2+(y–2)2 = 0, x2+y2+2x–4y+5 = 0 ___________(1)
(x–2)2+(y–3)2 = 0, x2+y2–4x–6y+13 = 0 ___________(2)
respectively
Equation of radical axis of (1) and (2) is
S1–S2 = 0
6x+2y–8 = 0
3x+y–4 = 0
Example 3
Find the equation of the circle which passes through the origin and belongs to the co-axial of
circles whose limiting points are (1,2) and (4,3)
Solution :
Equation of circles with limiting points (1,2) and (4,3) are
(x–1)2+(y–2)2 = 0 ⇒ x2+y2–2x–4y+5 = 0
(x–4)2+(y–3)2 = 0 ⇒ x2+y2–8x–6y+25 = 0
System of co-axial of circles equation is
x2+y2–2x–4y+5+ λ (x2+y2–8x–6y+25) = 0______________(1)
equation (1) passes through origin
∴ 5+25 λ = 0
∴ λ = –1/5
Substituting in (1) we get
4(x2+y2)–2x–14y = 0
2x2+2y2–x–7y = 0

97
98
CIRCLE - XI
Problem Solving

Example 1
Find the equation of the image of the circle x2+y2+16x–24y+183 = 0 by the line mirror
4x+7y+13 = 0
Solution :
The given circle and line are
x2+y 2+16x–24y+183 = 0_________(1) and 4x+7y+13 = 0_____________(2)
Centre and radius if the circle are
(–8, 12) and 64 + 144 – 183 = 25 = 5 respectively.

Equation of line C1C2 is 7x–4y+k = 0 it passes through (–8,12)


∴ –56–48+k = 0
k = 104
Equation of line C1C2 is 7x–4y+104 = 0__________(3)
To get the coordinates of M. Solve the equation
(2) & (3)
(2)×4+(3)×7
16 x + 28 y + 52 = 0
49 x – 28 y + 728 = 0
65x 
+ 780 = 0
x = –12
put the value of x in (2) we get
–48+7y+13 = 0
7y= 35
y=5
∴ coordinate of M is (–12,5)
M is the midpoint of C1and C2
h –8
∴ –12 = ⇒ h = –16
2
k + 12
5= ⇒ K = –2
2
∴ Equation of imaged circle is
(x+16)2+(y+2)2 = 25

99
x2+y2+32x+4y+235 = 0
2. The circle passing through the point (–1,0) and touching the y–axis at (0,2) also passes
through the point
⎛ –3 ⎞ ⎛ –5 ⎞ ⎛ –3 5⎞
(a) ⎜ ,10 ⎟ (b) ⎜ ,12 ⎟ (c) ⎜ , ⎟ (d) (–4,0)
⎝ 2 ⎠ ⎝ 2 ⎠ ⎝ 2 2⎠
Solution :
Family of circles passing through a point (0,2) and touching line x= 0 (y–axis) is
(x–0)2+(y–2)2+ λ x = 0
It passes through (–1,0)
∴ 1+4– λ = 0 y

∴ λ =5
∴ equation of circle is (0,2)

x2+y2+5x–4y+4 = 0 A B
x
o
If also passes through A(–x1,0) (–x ,0) (–1,0)
1

∴ x12–5x1+4 = 0
(x1–4)(x1–1)=0
x1= 4, x1= 1
∴ it also passes through A (–4,0)
3. Two parallel chords of a circle of radius 2 are at a distance 3 + 1 apart. If the chords subtend
π 2π
at the centre, angles of and , where k>0 then the value of [k] is...............
k k
( [k] denotes the greatest integer)
Solution :
π 1
Let = α = ∠ AOB = ∠ AOM
2k 2
Then ∠ CON = 2 α
In Δ AOM A M B
x
x 2 α α
cos α = 0 3+1–
2 2 2α 2α
In Δ CON 3+1– x
D
C N

3 +1 – x
cos2 α =
2
cos2 α = 2cos θ –1
2

x2
cos2 α = 2 –1
4
3 +1 – x x2
∴ = –1
2 2

100
3 + 1 – x = x2 – 2
x2+x– 3 – 3 = 0

∴ x=
(
–1± 1+ 4 3 + 3 )
2

– 1 ± 13 + 4 3
=
2

(
–1± 2 3 +1 )
=
2
( )
(∴ 13+4 3 = 2 3 + 1 2 )

– 1+ 2 3 +1
x=
2
x= 3
3 π
cos α = = cos
2 6
π
α=
6
π π
∴ Required angle =
k
= 2α =
3 ⇒ k=6 thus [k] =6

4. Let ABC and AB C′ be two non-congruent triangles with sides AB = 4, AC = A C′ = 2 2 and


angle β = 30°. The absolute value of the difference between the areas of these triangles is
Solution :
Draw circle through AC C′ and AB intersect the circle and P
1
C'
AD 2
D
In Δ ABD = = sin300 2
2
AB 2 2
C
2 2 30°
B
AD 1 A' 4
= P
4 2
∴ AD = 2 = DC = C′ D
Difference of areas of Δ ABC and Δ AB C′ is Δ AC C′
1
∴ ar( Δ AC C′ ) = ×4×2 = 4 sq.u.
2
4. The centres of two circles C1 and C2 each of unit radius are at a distance of 6 units from each
other. Let P be the mid-point of the line segment joining the centres of C1 and C2 and C be a
circle touching circles C1 and C2 externally. If a common tangents to C1 and C passing through P
is also a common tangent to C2 and C1 then the radius of the circle C is

101
Solution :
In Δ CPC2 6
2 2 2
CP = (CC2) –(C2P)
k2 = (r+1)2 –9 P
3
2 2
k =r +2r–8_________________(1) C
1 1 C
8 2

In Δ PQC2 k
2 2 2
PQ = 3 –1 r Q

=8 C

∴ In Δ CPQ
k2 = r2+8______________(2)
From (1) & (2)
r2+8 = r2+2r+2r–8
16 = r
∴ r=8
5. A straight line through the vertex P of a triangle PQR intersects the side QR at the point S and the
circumcircle of the triangle PQR at the point T. If S is not the centre of the circumcircle then
1 1 2 1 1 2
(a) + < QS.SR (b) + > QS.SR
PS ST PS ST

1 1 4 1 1 4
(c) + < (d) + >
PS ST QR PS ST QR
Solution : P

Points P, Q, T, R are concyclic


∴ PS.ST = QS.SR
PS + ST S
⇒ ≥ PS.ST (AM ≥ GM) R
2 Q 3

∴ PT ≥ 2 PS.ST T

1 1 2 2
PS ST ≥
and + = QS.SR (Dividing by PS.ST)
PS.ST
SQ + SR
Also,
2 ≥ SQ.SR (AM ≥ GM)

QR

2 ≥
SQ.SR
1 2
⇒ SQ.SR ≥ QR
2 4
⇒ SQ.SR ≥ QR

102
1 1 2 4
+
PS ST ≥ QS.SR QR≥
6 Let ABCD be a quadrilateral with area 18, with side AB parallel to the side CD and
AB = 2CD. Let AD be perpendicular to AB and CD. If a circle is drawn inside the quadrilateral
ABCD touching all the sides then its radius is
(a) 3 (b) 2 (c) 3/2 (d) 1.
Solution :
ABCD is a trapezium ( AB parallel to CD)
1
∴ ar(ABCD) = ×h (sum of parallel sides)
2
1
= ×2r(2a+a)
2 Y
18 = r×3a D C (a,2r)
(o,2r)
ar = 6
CB is a tangent to the circle
(r,r)
∴ equation of tangent is
r R
– 2r
y= (x–2a) ⇒ 2rx+ay–4ar = 0
a OA B
(2a,0)
It is a tangent to the circle (x–r)2+(y–r)2 = r2

2r 2 + ar – 4ar
∴ r=
4r 2 + a 2
2
r 4r 2 + a 2 = 2r –3ar

4r 2 + a 2 = 2r–3a
Squaring
4r2+a2=4r2+9a2–12ar
12r = 8a ⇒ 3r = 2a
ar = 6
29
r=
3
2a 2
=6
3
a2 = 9
a= ±3
∴ r=2
7 The radius of the least circle passing through the point (8,4) and cutting the circle
x2+y2 = 40 orthogonally is
(a) 5 (b) 7 (c) 2 5 (d) 3 5

103
Solution :
Let the circle be x2+y2+2gx+2fy+c = 0___________(1)
Given circle is x2+y2 = 40 _____________________(2)
These two circles are orthogonal
∴ c–40 = 0 ⇒ c = 4 0
(1) passes through (8,4)
64+16+16g+8f+40 = 0
120+16g+8f = 0
f+2g+15 = 0 or f = –(2g+15)
radius = g 2 + f 2 – c

= g 2 + (2g + 15) 2 – 40
For least circle radius must be minimum
Let f(g) = g2+(2g+15)2 – 40 is minimum
f ′ (g) = 2g+4(2g+15) = 0
10g = –60
g = –6
f ′′ (g) = 10>0 minimum
f = –(–12+15) = –3
Equation of circle in x2+y2–12x–6y+40 = 0
radius = 36 + 9 – 40
= 5
8 P is a point (a,b) in the first quadrant. If the two circles which pass throngh P and touch both the
co-ordinate axes cut at right angles, then
(a) a2–6ab+b2 = 0 (b) a2+2ab–b2 = 0
(c) a2–4ab+b2 = 0 (d) a2–8ab+b2 =0
Solution :
Equation of the two circles be
(x–r)2+(y–r)2 = r2 ⇒ x2+y2–2xr–2yr+r2 =0
These two circles passes through (a,b)
∴ (a–r)2+(b–r)2 = r2
a2+r2–2ar +b2+r2–2br–r2 = 0
r2–2r(a+b)+(a2+b2) = 0
It is a quadratic equation in r
∴ r1+r2 = 2(a+b)and r1r2 = a2+b2
Condition for orthogonality is
2g1g2+2f1f2 = C1+C2
2r1r2+2r1r2=r12+r22
4r1r2 = r12+r22
6r1r2 = r12+r22+2r1r2
6r1r2=(r1+r2)2
6(a2+b2) = 4 (a+b)2

104
6a2+6b2= 4a2+4b2+8ab
2a2+2b2–8ab = 0
a2+b2–4ab = 0
9 A circle S ≡ 0 passes throngh the common points of family of circles x2+y2+dx–4y+3 = 0( λ ∈ R)
and have minimum area then
(a) area of S ≡ 0 is π squ.
(b) radius of director circle of S ≡ 0 is 2
(c) Radius of director circle of S ≡ 0 for x–axis is 1 unit
(d) S ≡ 0 never cuts |2x| = 1
Solution :
(x2+y2–4y+3)+ λ x = 0
∴ x= 0 and y2–4y+3 = 0
(y–3)(y–1) = 0
y=3,1
∴ (0,3) (0,1) are common points.
x2+y2+2gx+2fy+c = 0
passion through (0,3) & (0,1)
9 + 6f + c = 0 ___________________(1)
1 + 2f + c = 0 ___________________(2)
(1) – (2) we get
8 + 4f = 0
f = –2DQGc = 3
∴ x2+y2+2gx–4y+3 = 0
radius = g2 + 4 – 3 = g2 +1
for minimum area radius must be minimum
Since g2+1 is positive so g must be zero
∴ radius = 1
Area = π r2 = π sq.u.
Radius of director circle is 2 times the radius of the given circle.
∴ Radius of director circle is 2
10 Area of part of circle x 2+y 2–4x–6y+12 = 0 above the line 4x+7y–29 = 0 is Δ ,
then [ Δ ] = ________ [.] is greatest integer function.
Solution
Since line 4x+7y–29 =0
passes through the centre (2,3) of the circle
∴ The line is a diameter of a circle with radius 4 + 9 – 12 = 1
1 2 4x+7y-29=0
∴ area of semi circle is = πr (2,3)
2
1
= π
2

105
3.14
= =1.57
2
Hence[ Δ ] = [1.57] = 1
PRACTICE QUESTIONS

π
1. Let 0 < α < be a fixed angle. If P = (cos θ, sin θ) and Q = ( cos(α – θ ) , sin (α – θ ) Q is obtained
2
from P by
a. clockwise rotation around origin through an angle α
b. anti-clockwise rotation around origin through an angle α
c. reflection in the line through origin with slope tan α
d. reflection in the line through origin with slope tan α /2
2. If the tangent at the point P on the circle x2+y2+6x+6y = 2 meets the straight line 5x–2y+6 = 0
at a point Q on the y-axis, then the length of PQ is
a. 4 b. c. 5 d. 3 5
3. The equations to the sides AB, BC, CA of a ΔABC are drawn on AB, BC, CA as diameters.
The point of concurrence of the common chord is
a. centroid of the triangle b. orthocenter
c. circumcentre 2 5 d. incentre
4. The number of rational point(s) (a point (a, b) is rational, if a and b both are rational numbers)
on the circumference of a circle having centre (π, e) is
a. at most one b. at least two c. exactly two d. infinite
2 2
5. The locus of a point such that the tangents drawn from it to the circle x +y –6x–8y = 0 are
perpendicular to each other is
a. x2+y2–6x–8y–25 = 0 b. x2+y2+6x–8y–5 = 0
2 2
c. x +y –6x+8y–5 = 0 d. x2+y2–6x–8y+25 = 0
6. If the two circles x2+y2+2gx+2fy = 0 and x2+y2+2g1x+2f1y = 0 touch each other, then
a. f1g = fg1 b. ff1 = gg1
c. f2+g2 = f12 + g12 d. none of these
7. The number of integral values of λ for which x2+y2+ λ x+(1– λ )y+5 = 0 is the equation of a
circle whose radius cannot exceed 5, is
a. 14 b. 18 c. 16 d. none of these
2 2
8. The circle x +y +4x–7y+12 = 0 cuts an intercept on y-axis of length
a. 3 b. 4 c. 7 d. 1
2 2
9. One of the diameter of the circle x +y –12x+4y+6 = 0 is given by
a. x+y = 0 b. x+3y = 0 c. x=y d. 3x+2y = 0
10. The coordinates of the middle point of the chord cut off by 2x–5y+18 = 0 by the circle
x2+y2–x+y2–54 = 0 are
a. (1, 4) b. (2, 4) c. (4, 1) d. (1, 1)

106
11. A variable chord is drawn through the origin to the circle x2+y2–2ax = 0. The locus of the
centre of the circle drawn on this chord as diameter is
a. x2+y2+ax = 0 b. x2+y2+ay = 0
2 2
c. x +y –ax = 0 d. x2+y2–ay = 0
12. If O is the origin and OP, OQ are distinct tangents to the circle x2+y 2+2gx+2fy+c = 0, the
circumcentre of the triangle OPQ is
a. (–g, –f) b. (g, f) c. (–f, –g) d. none of these
2 2
13. Equation of the normal to the circle x +y –4x+4y–17 = 0 which passes through (1, 1) is
a. 3x+2y–5 = 0 b. 3x+y–4 = 0
c. 3x+2y–2 = 0 d. 3x–y–8 = 0
14. The equation of the circle touching the lines y = x at a distance 2 unit from the origin is
a. x2+y2–4x+2 = 0 b. x2+y2+4x–2 = 0
c. x2+y2+4x+2 = 0 d. none of these
15. The shortest distance from the point (2, –7) to the circle x2+y2–14x–10y–151 = 0 is
a. 1 b. 2 c. 3 d. 4
2 2
16. The equation of the image of the circle (x–3) +(y–2) = 1 by the mirror x+y = 19 is
a. (x–14)2+(y–13)2 = 1 b. (x–15)2+(y–14)2 = 1
c. (x–16)2+(y–15)2 = 1 d. (x–17)2+(y–16)2 = 1
2 2
17. If P and Q are two points on the circle x +y –4x–4y–1 = 0 which are farthest and nearest
respectively from the point (6, 5), then
22 22 19
a. P= – ,3 b. Q= ,
5 5 5
14 11 14
c. P= ,– d. Q = – ,–4
3 5 3
18. A circle of the coaxial system with limiting points (0, 0) and (1, 0) is
a. x2+y2–2x = 0 b. x2+y2–6x+3 = 0
2 2
c. x +y = 1 d. x2+y2–2x+1 = 0
19. If a variable circle touches externally two given circles, then the locus of the centre of the
variable circle is
a. a straight line b. a parabola
c. an ellipse d. a hyperbola

PASSAGE – 1
Let A ≡ (a ,0) and B be two fixed points and P moves on a plane such
that PA = nPB
On the basis of above information, answer the following questions:
20. If n ≠ 1 , then the locus of a point P is
a. a straight line b. a circle c. a parabola d. an ellipse
21. If n = 1, then the locus of a point P is
a. a straight line c. a circle c. a parabola d. a hyperbola

107
22. If 0 < n < 1, then
a. A lies inside the circle and B lies outside the circle
b. A lies outside the circle and B lies inside the circle
c. both A and B lies on the circle
d. both A and B lies inside the circle
23. If n > 1, then
a. A lies outside the circle and B leis inside the circle
b. A lies outside the circle and B leis inside the circle
c. both A and B lies on the circle
d. both A and B lies inside the circle
24. If focus of P is a circle, then the circle
a. passes through A and B
b. never passes through A and B
c. passes through A but does not pass through B
d. passes through B but does not pass through A

PASSAGE – 2
For each natural number k, let Ck denotes the circle with radius k units and centre at the origin.
On the Ck-, a particle moves k units in the counter clockwise direction. After completing its
motion on Ck, the particle moves to Ckt1in some well defined manner, where k>0.The motion
of the particle continues in this manner.
On the basis of above information, answer the following questions:
25. Let, K=1 the particle starts at (1, 0). If the particle crossing the positive direction of the x-axis
for the first time on the circle Cn, then n is equal to
a. 3 b. 5 c. 7 d. 8
26. If k ∈ N and , the particle starts (–1, 0) the particle cross x-axis again at
a. (3,0) b. (1,0) c. (4,0) d. (2,0)
27. If and , the particle moves in the radial direction from circle Ck to CK+1. If particle
starts form the point (–1, 0), then
a. it will cross the +ve y-axis at (0, 4)
b. it will cross the –ve y-axis at (0, –4)
c. it will cross the +ve y-axis at (0, 5)
d. it will cross the –ve y-axis at (0, –5)
28. If and , particle moves tangentially form the circle Ck to Ck+1, such that the length
of tangent is equal to k units itself. If particle starts form the point (1, 0), then
a. the particle will cross x-axis again at x = 3
b. the particle will cross x-axis again at x = 4
c. the particle will cross +ve x-axis again at x =
d. (
the particle will cross +ve x-axis again at x ∈ 2 2 ,4 )
29. Let the particle starts from the point (2, 0) and moves π / 2 units, on circle C2 in the counter-
clockwise direction, then moves on the circle C3 along the tangential path, let this straight line
(tangential path traced by particle) intersect the circle C3 at the points A and B tangents drawn

108
at A and B intersect at

a.
9
;
2 2 2 2
9
b. (9 2 ,9 2 ) c. (9, 9) d. ( 2, 2 )
Match Type:
30. Observe the following columns:
Columns I Columns II
a. If the shortest and largest distance from the point p. M+L = 10
(10, 7) to the circle x2+y2–4x–2y–20 = 0 are L
and M respectively, then
b. If the shortest and largest distance from the point Q. M + L = 20
(3, –6) to the circle x2+y2–16x–12y–125 = 0 are L
and M respectively, then
c. If the shortest and largest distance from the point r. M + L = 30
(6, –6) to the circle x2+y2–4x+6y–20 = 0 are L s. M – L = 10
and M respectively, then t. M – L = 26

31. Observe the following columns:


Columns I Columns II
a. If the straight lines y = a1x+b and p. a12 + a 22 = 4
y = a2x+b (a1 ≠ a 2 ) meet the
coordinate axes in concyclic points, then
b. If the chord of contact of the tangents drawn q. a1+a2 = 3
to x2+y2= b2 from any point on x2+y2 = ,
touches the circle
x2+y2 = a 22 (a1 ≠ a 2 ) , then
c. If the circles x2+y2+2a1x+b = 0 and r. a 1a 2 = b
x2+y2+2a2x+b = 0 (a1 ≠ a 2 ) and cuts s. a 1a 2 = 1
orthogonally, then t. a 1a 2 = b 2

ANSWERS

1. d 2. c 3. b 4. a 5. a 6. a
7. c 8. d 9. b 10. a 11. c 12. d 13. b
14. a 15. b 16. d 17. b 18. d 19. d 20. b
21. a 22. a 23. b 24. b 25. c 26. c 27. c
28. d 29. a
30. a q,s b → r,t c→ s
31. a → p,q,s b → p,q,s,t c → p,q,r,s

109
110
PARABOLA - I

111
112
a ahg
⇐ Δ = hbf
gtc
113
PRACTICE QUESTIONS

114
115
116
PARABOLA - II
PARABOLA
A parabola is the locus of a point, whose distance from a fixed point is equal to the perpendicular
distance from a fixed straight line.

Let S be the focus, ZZ ′ be the directrix.


Consider S (a, 0) and equation of ZZ ′ is x+a = 0.
Axis of parabola is x-axis.
Now according to definition.
PS = PM
x+a
(x – a )2 + y 2 =
1 S (a,0)

(x – a )2 + y 2 = (x + a )2
y 2 = 4ax
Vertex (0, 0)
Tangent of latus rectum x = 0
Extremities of latus rectum (a, 2a) , (a, –2a)
Length of latus rectum.=4a
Focal distance (SP) SP = PM = x + a
2
Parametric form x = at , y = 2at, t is parameter.
Focal distance – the distance of a point on the parabola from the focus.
Focal chord – A chord of the parabola, which passes through the focus.
Double ordinate – A chord of the parabola perpendicular to the axis of the parabola.
Latus Rectum – A double ordinate passing through the focus or a focal chord perpendicular to
the axis of parabola.
• Perpendicular distance from focus on directrix = half the latus rectum.
• Vertex is middle point of the focus and the point of intersection of directrix and axis.
• Two parabolas are said to be equal if they have the same latus rectum.

Other Standard Forms of Parabola:

S A

Equation of curve: y2 = – 4ax x2= 4ay x2 = –4ay


Vertex (0, 0) (0, 0) (0, 0)
117
Focus (–a, 0) (0, a) (0, –a)
Directrix x–a=0 y+a=0 y–a=0
Equation of axis y=0 x=0 x=0
Tangent of vertex x=0 y=0 y=0
Parametric form (–at2, 2at) (2at, at2) (2at, –at2)

Position of a point with respet to Parabola


y2 = 4ax y2 = – 4ax x2 = 4ay x2 = – 4ay

P (x1,y1)

S1 = y12 – 4ax 1 S1 = y12 + 4ax1 S1 = x12 – 4ay1 S1 = x12 + 4ay1


S1< 0 → Inside S1< 0 → Inside S1< 0 → Inside S1< 0 ← inside
S1> 0 → Outside S1> 0 → Outside S1> 0 → Outside S1> 0 → Outside
S1= 0 → on the parabola S1= 0 → on the parabola S1= 0 → on the parabola S1= 0 → on the parabola
y
Equation of Parabola when vertex is shifted. z

I. Axis is Parallel to x-axis: A S


Let vertex A be (p, q) then equation of parabola be (p,q)
(y – q)2 = 4a (x – p). O
x

z'

S
II. Axis is parallel to y-axis:
Let vertex A be (p, q) then equation of parabola is A (p,q)
z z'
(x – p)2 = 4a (y –q) O
x

Example: 1 The equation of parabola is y = ax2 + bx + c, find its vertex, focus, directrix,
Equation of a is, length of latus rectum.
b b2 b2
y=a x + x+ 2 – 2 +c
2

a 4a 4a
2
b b2
y–c=a x+ –
2a 4a 2

118
2
b 1 b2
x+ = y+ –c
2a a 4a
b b 2 – 4ac 1
X = 4 AY where X = x + , Y = y +
2 ,4A =
2a 4a a

b b 2 – 4ac
Vertex : X = 0, Y = 0 i.e. – , –
2a 4a

b 1 + 4ac – b 2
Focus: X = 0, Y = A i.e. – ,
2a 4a

b 2 – 4ac + 1
Equation of directrix: y + =0
4a
b
Equation of axis: x + =0
2a
1
Length of latus rectum =
a
Example: 2 The equation of parabola is y2 = ax + ay. Find its vertex, focus, directrix, axis
and length of latus rectum.
y2 – ay = ax

2
a2 a2
y – ay + = ax +
4 4
2
a a
y– =a x+
2 4
Y2 = 4AX
a a a
Where: Y = y – , X = x + , 4A = a ie. A =
2 4 4
a a
Vertex – ,
4 2
a
Focus 0,
2
a
Directrix x + =0
2

119
a
Axis y – =0
2
Length of latus rectum = a
PRACTICE QUESTIONS
1. The equation of parabola whose focus is at (–1,–2) and directrix is x–2y+3 = 0 is
a. 4x2–y2–4xy+4x–32y–16 = 0 b. x2+4y2+4xy+x+6y+16 = 0
c. 4x2+y2+4xy+4x+32y+16 = 0 d. 4x2+y2–4xy+4x–32y–1 = 0
2. The equation of parabola whose vertex is at (4, –1) and focus is (4, –3) is
a. y2–8x+8y+24 = 0 b. x2–8x+8y+24 = 0
c. y2–8x–8y+24 = 0 d. x2+8x–8y–24 = 0
2
3. The focal distance of a point on the parabola y =8x is 8, then coordinates of the point (s) is/are
a. (
4 3 ,6 ) b. (
6,4 3 ) c. (
4 3 ,–6 ) d. (
6,–4 3 )
4. The equation of the parabola whose focus is (0, 0) and tangent at the vertex is x–y+1 = 0 is
a. x2+y2+2xy–4x+4y–4 = 0 b. x2+y2+4xy+4x+4y+4 = 0
c. x2+y2–4xy+4x+4y–4 = 0 d. x2+y2–4xy–4x–4y–4 = 0

ANSWERS

1. c 2. b 3. b, d 4. a

120
PARABOLA - III
PARAMETRIC FORM:
y2 = 4ax y2 = – 4ax x2 = 4ay x2 = – 4ay
x = at2 x = – at2 x = 2at x = 2at
y = 2at y = 2at y = at2 y = – at2
Parametric (at2, 2at) (– at2, 2at) (2at, at2 ) (2at, – at2 )
coordinates

Properties of Focal chord:


1. If the chord joining P(t1) and Q(t2) is the focal chord then t1.t2 = – 1.
P, S and Q lies on the focal chord
∴ P, S and Q are collinear y
slope of PS = slope of SQ
2at1 2at P 2
2
= 2 2 (at1 ,2at1)
at1 – a at 2 – a
S (a,o)
x
O
2 t1 2t
2
= 2 2 (at 2,2at2)
t1 – 1 t 2 – 1 Q 2

t1t 22 – t1 = t 2 t12 – t 2
t 2 – t1 = t1 t 2 ( t1 – t 2 )
1
– 1 = t 1t 2 ort 2 = –
t1

a – 2a
Extremities of a focal chord are (at2, 2at) and , .
t2 t
2
1
2. Length of focal chord is a t + y
t
PQ = PS + SQ P 2
(at,2)at
a
= at2 + a + +a S (a,o)
t2 O
x
a − 2a
1 ,
= a t + 2 +2
2
Q
t2 t

t
2
1
= a t+
t
3. The length of the focal chord which makes an angle θ with the positive direction of x-axis is
4a cosec2 θ .
2
1
We know PQ = a t +
t
121
2a
2at +
slope = tan θ = t
a
at 2 – 2 y
t

2 P 2
slope = tan θ = (at ,2at)
1
t– S θ
t x
O
1 a − 2a

2cot θ = t –
,
t2 t
Q
t
2
1
∴ PQ = a t +
t
2
1
=a t– +4
t
= a [4 cot2 θ + 4]
= 4a cosec2 θ ( cos ecθ ≥ 1 )
• Minimum length of PQ = 4a (i.e.latusrectum )
4. Semi latus rectum of a parabola is the harmonic mean between the segments of any focal chord
of the parabola.
a y
SP = a + at2, SQ = a +
t2
P 2
1 1 1 1 1 t2 1 (at ,2at)
+ = + = + =
SP SQ a + at 2
a + 2 a + at
a 2
at + a a
2
S
x
t O
a − 2a
,

SPxSQ
t2 t
Q
∴ 2a = 2x
SP + SQ
Semi latus rectum = Harmonic Mean of SP and SQ.
y
5. Circle described on the focal length as diameter touches the
tangent at vertex.
Equation of circle PS as diameter is
(x – at2) (x – a) + (y – 2at)y = 0 x
Equation of y-axis is x = 0 (a,0)
After solving y2 – 2aty + a2t2 = 0
(y – at)2 = 0
∴ circle touches the y-axis at (0, at).

122
Example: 1 The length of a focal chord of the parabola y2 = 4ax at a distance b from the
vertex is c, then

a. b2 = 4ac b. b 2c = a 3 c. b2c = 4a 3 d. 4b2c = a3


Solution: PQ = 4a cosec2 θ = c
y
b
In ΔOMS , sin θ =
a P

a2
cosec2 θ = S θ
b2 (a,0) x
O
b M
a2 Q
∴ c = 4a. 2 b2c = 4a 3
b
Ans: c

Example: 2 The coordinates of the ends of a focal chord of a parabola y2 = 4ax are (x1, y1)
and (x2, y2) then value of x1 x2 + y 1 y2 is equal to

a. 3a 2 b. – 3a 2 c. a2 d. – a2
( ) ( )
Solution: Let at12 ,2at1 ≡ (x1 , y1 ) and at 22 ,2at 2 ≡ (x 2 , y 2 ) such that t1t2 = –1

∴ x1x2 + y1y2 = a t1 t 2 + 4a t1t 2 = a – 4a = –3a


2 2 2 2 2 2 2

Ans: b
PRACTICE QUESTIONS
1. The focus of the parabola x2+8x+12y+4 = 0 is
a. (4, 2) b. (–2, –4) c. (2, 4) d. (–4, –2)
2. The equation of the parabola with vertex at (3, 2) and focus at (5, 2) is
a. x2–8x–4y–28 = 0 b. y2–8x–4y–28 = 0
2
c. x +8x–4y–28 = 0 d. y2+8x+4y–28 = 0
3. The equation of the latus rectum of the parabola x2+4x+2y = 0 is
a. 2y–3 = 0 b. 3y–2 = 0
c. 2y+3 = 0 d. 3y+2 = 0
4. The equation of the parabola whose axis is parallel to x-axis and which passes through the points
(0, 4), (1, 9) and (–2, 6) is
a. y2+5x–25y+139 = 0 b. 3y2+5x–25y+52 = 0
2
c. 2y –5x–25y+68 = 0 d. none of these
5. The parametric equation x = at2+bt+c, y = a ′t 2 + b′t + c′ represents
a. a circle b. a parabola c. an ellipse d. none of these

ANSWERS
1. d 2. b 3. a 4. c 5. b

123
124
PARABOLA - IV
Line and Parabola:
y y y

x x O x
O O

Let equation of line be y = mx + c and equation of parabola be y2 = 4ax.


(mx + c)2 = 4ax
m2x2 + 2x(mc – 2a) + c 2 = 0
D = {2(mc – 2a )}2 – 4.m 2 .c 2
= 4(4a2 – 4amc)
If D < 0, line do not intersect parabola.
i.e. a < mc
If D = 0 i.e. a = mc, line touches the parabola (condition of tangency)
If D > 0 i.e. a > mc, line intersect the parabola at two points.
Equation of tangent (Point form)
y
Equation of parabola y2 = 4ax
Differentiate w.r.t.x
dy
2y = 4a (x1,y1)
dx x
O
2a
slope of tangent =
y1

2a
Equation of tangent y – y1 = y (x – x1)
1
y
y y1 – y12 = 2ax – 2ax1
y y1 = 2ax + 2ax1
y y1 = 2a (x+x1) (at2,2at)
x
O
Equation of tangent (Paramatric form)
y.2at = 2a (x + at2)
ty = x + at2

125
Equation of tangent (slope form)
a
y = mx +
m
a 2a
Point of contact ,
m2 m
• Equation of tangent to the parabola (y – k)2 = 4a (x – h) is
a
y – k = m(x – h) +
m

Equation of tangent:
Parabola Point form Pt. of contact Parametric form Pt. of contact slope form Pt. of contact
a a 2a
y2=4ax yy1=2a(x + x1) (x1, y1) ty = x + at2 (at2, 2at) y = mx + ,
m m2 m

a a – 2a
y2= –4ax yy1=–2a(x + x1) (x1, y1) ty = – x+ at2 (– at2, 2at) y = mx – – ,
m m2 m

a 2a a
x2= 4ay xx1= 2a(y + y1) (x1, y1) tx = y + at2 (2at, at2) x = my + ,
m m m2

a – 2a – a
x2= – 4ay xx1=–2a(y + y1) (x1, y1) tx = – y + at2 (2at, – at2) x = my – ,
m m m2

Pair of Tangents from point (x 1, y 1 )


Let eqn of parabola be y2 = 4ax A
S ≡ y2 – 4ax
2
S1 ≡ y1 – 4ax1 P
T ≡ yy1 – 2a(x+x1) (x 1, y 1)
Equation of pair of tangents is SS1 = T2 i.e.
( )
(y2 – 4ax) y12 – 4ax1 = {yy1 – 2a (x + x 1 )}
2 B

Properties of Tangents: P(t1)


1. Point of intersection of two tangents of the parabola:-
Equation of tangent at P is t1y = x + at 12
A
Equation of tangent at Q is t2y = x + at 22
Solving these equations, we get
x = at1t2, y = at1 + at2 Q(t2)
A(at1t2, a(t1 + t2))
2. Locus of foot of prependicular from focus upon any tangent is tangent at vertex:-
Equation of tangent at P is ty = x + at2

126
Let the tangent meet y-axis at Q then Q(0, at)
y
– at
∴ slope of QS = = –t
a
1 Q P(at2,2at)
slope of tangent =
t x
1 S(a,0)
x (– t) = –1 SQ ⊥ tangent
t
3. Length of tangent between the pt. of contact and the point
where tangent meets the directrix subtends right angle at focus:-
Eqation of tangent at P(t)
ty = x + at2
Point of intersection with directrix x = – a is
a
– a , at –
t
2at 2t
slope SP = 2
= 2
at – a t – 1
a
at – 2
slope QS = t = t –1
– 2a – 2t
m1 m2 = –1
PS ⊥ QS.
4. Tangent at extremities of focal chord are perpendicular and intersect on directrix
(Locus of intersection point of tangents at extremities of focal chord is directrix)
a – 2a
Let P(at2, 2at) and Q ,
t2 t
Equation of tangent at P ty = x+at2 ......(1)
1 a
Equation of tangent at Q – y=x+ 2
t t
a
y = – tx – ......(2)
t
Point of intersection of both tangents, we get after sloving (1) & (2) i.e.
x+a = 0
A point lies on the directrix.

127
PRACTICE QUESTIONS

1. If the tangents to the parabola y2 = 4ax at the points (x1, y1) and (x2, y2) meet at the point (x3, y3)
then

a. y 32 = y1y 2 b. 2 y3 = y1 + y 2 c. d. none of these

2. A right angled triangle ABC is inscribed in parabola y2 = 4x, where A is vertex of parabola and
∠BAC = 900. If AB = , then area of ΔABC is
a. 40 b. 10 c. 20 d. 4 5
3. ( )
The locus of the point 3h , 3k + 2 if it lies on the line x–y–1 = 0 is a
a. circle b. parabola c. straight line d. none of these
4. The length of the chord of the parabola y2 = x which is bisected at the point (2, 1) is
a. 5 b. 4 c. 2 5 d. 5 2
2
5. If y = mx+c touches the parabola y = 4a (x+a), then
a a 2a a
a. c= b. c=a+ c. c= d. c = am +
m m m m

25 1 1
= +
y3 y1 y 2

ANSWERS
1. a 2. c 3. b 4. c 5. d

128
PARABOLA - V

Example: 1 The focal chord to y2 = 16x is tangent to (x–6)2 +y2 = 2, then the possible value of
the slope of this chord, are
1 1
a. {–1, 1} b. {–2, 2} c. – 2, d. 2,–
2 2
Solution: The focus of parabola is (4, 0). Let slope of focal chord be m. Equation of focal
chord is y = m(x–4). It is tangent to the circle then
6m – 4m
= 2
m2 + 1
4m2 = 2(m2+1)
2m2 = 2
m = ±1
Ans: a

Example: 2 The curve represented by ax + by = 1 , where a, b > 0 is


a. a circle b. a parabola c. an ellipse d. a hyperbola
ax = 1 – by
Solution: ax = 1+by – 2 by

(
(ax–by–1)2 = – 2 by )
2

a2x2+b2y2+1–2abxy–2ax+2by = 4by
a2x2–2abxy+b2y2–2ax–2by+1 = 0

a  – ab– a
Δ = – abb  – b
– a– b1 = a2(b2–b2)+ab(–ab–ab) –a(ab2+ab2)
= 0 – 2a2b2–2a2b2
= –4a 2 b 2 ≠ 0
h2–ab = (–ab) 2 – (a2) (b2) = 0
∴ It is a parabola
Ans: b

129
Example: 3 The equation of the directrix of the parabola y2+4x+4y+2 = 0 is
–3 3
a. x = –1 b. x=1 c. x= d. x=
2 2
Solution: y2 + 4y = –4x–2
1
(y+2)2 = –4 x –
2
y2 = –4AX
1
Equation of directrix is X = A i.e. x – =1
2
3
2x–3 = 0 or x =
2
Ans: d

Example: 4 The locus of the midpoint of the segment joining the focus to a moving point on the
parabola y2 = 4ax is another parabola with directrix
a. y=0 b. x = –a c. x=0 d. none of these

Solution: Let P(at2, 2at) lies on the parabola


y2 = 4ax
Mid point of PS is Q.
at 2 + a 0 + 2at
h= ,k =
2 2
2h – a k
= t2, = t
a a
2
2h – a k
=
a a
a(2h – a) = k2
a
Locus of (h, k) is y2 = 2a x –
2
a a
Equaiton of directrix is x – =–
2 2
x=0
Ans: c

130
Example: 5 The angle between the tangents drawn from the point (1,4) to the parabola y2 = 4x
is
π π π π
a. b. c. d.
6 4 3 2
2
Solution: Equation of tangent of the parabola y = 4x is
1
y = mx +
m
This equation passes through (1, 4) i.e.
1
4= m+
m
2
m – 4m+1 = 0
m1.m2 = 1 and m1 + m2 = 4
m1 – m 2
Angle between the two tangents is tan θ =
1 + m1m 2

tan θ =
(4)2 – 4 =
12
= 3
1+1 2

π
∴θ =
3
Ans: c
Exercise: 6 Tangent to the curve y = x2+6 at a point (1, 7) touches the circle x2+y2+16x+12y+c
= 0 at a point Q then coordinates of Q are
a. (–6, –11) b. (–9, –13) c. (–10. –15) d. (–6, –7)

Solution: Equation of tangent at (1, 7) to the curve y = x2+6 is


y+7
= x+6
2
2x – y+5 = 0 .....(1)
This line also touches the circle i.e.
Equation of normal of circle passing through
centre (–8, –6).
x+2y + λ = 0
–8 –12 + λ = 0
λ = 20
∴ x + 2 y + 20 = 0 .....(2)
Q is intersection point of (1) and (2)
x = –6, y = –7
Q(–6, –7)
Ans: d
131
Exercise: 7 Consider the two curves c1: y2 = 4x, c2: x2+y2–6x+1 = 0. Then
a. c1 and c2 touch each other only at one point.
b. c1 and c2 touch each other only at two points.
c. c1 and c2 intersect (but do not touch) at exactly two points.
d. c1 and c2 neither intersect nor touch each other.

Solution: Let eqn of tangent of parabola be


1
y = mx + is also a tangent to the circle then
m

1
3m +
m =2 2
1 + m2

(3m 2
+1 )
2

(
= 8 1 + m2 )
2
m
m –2m2+1 = 0
4

m2 = 1 m = ± 1 Two common tangents are possible.

Exercise: 8 If b, c are intercepts of a focal chord of the parabola y2 = 4ax then c is equal to
b a ab ab
a. b–a b. b–a c. a–b d. b–a
2 xSAxSB
Solution: We know that 2a =
SA + SB
2 bc
2a = ⇒ ab = bc − ac
b+c
ab = ( b − o ) c

ab+ac = bc
ab
=c
b–a
Ans: d
Exercise: 9 The circle x2+y2–2x–6y+2 = 0 intersects the parabola y2 = 8x orthogonally at the
point P. The equation of the tangent to the parabola at P can be
a. 2x –y+1 = 0 b. 2x+y –2 = 0
c. x+y –4 = 0 d. x –y –4 = 0
2
Solution: Let y = mx + be tangent to y2 = 8x. Since circle intersects the parabola
m
orthogonally. So this tangent is the normal for the circle. Every normal of the circle passes
through its centre. So centre (1, 3).
132
2
3= m+ m 2 – 3m + 2 = 0
m
(m – 2) (m – 1) = 0
m = 1, 2.
y = x+2 or y = 2x+1
Ans: a

133
134
PARABOLA - VI

Equation of Normal
(i) Point form :
y2 = 4ax
Tangent
Differentiate w.r.t.x
dy P(x1,y1)
2y = 4a
dx
Normal
dy 2a
=
dx y
y1
Slope of normal = –
2a
Equation of normas at (x1,y1) is
y1
y–y1 = – (x–x1)
2a
(ii) Parametric form : P(at2,2at)
replace x 1 by at2 and y1 by 2at
2at
y–2at = – (x–at2)
2a
y = –tx+at3+2at
tx+y–at3–2at = 0
(iii) Slope form:
Replace t by – m
y = mx–2am–am3
y = mx+c is normal to parabola y2 = 4ax if
c = –2am–am3 ie., condition of normal.
Equation of Normal
Parabola Point form Pt.of contact Parametric form Point of contact slope Form Pt.of contact
– y1
y2=4ax y–y1= (x–x1) (x1,y1) y=–tx+2at+at3 (at2,2at) y=mx–2am–am3 (am2,–2am)
2a
y1
y2= –4ax y–y1= (x–x1) (x1,y1) y=–tx+2at+at3 (–at2,2at) y=mx+2am+am3 (–am2,2am)
2a
x1 a ⎛ 2a a ⎞
x2=4ay x–x1=– (y–y1) (x1,y1) x= –ty+2at+at3 (2at,at2) y=mx+2a+ ⎜– , 2 ⎟
2a m2 ⎝ m m ⎠
x1 a ⎛ 2a a ⎞
x2=–4ay x–x1=
(y–y1) (x1,y1) x= ty+2at+at3 (2at,–at2) y=mx–2a– ⎜ ,– 2 ⎟
2a m2 ⎝m m ⎠
Equation of normal to the parabola (y–k)2= 4(x–h) is
y–k = m(x–h) –2am –am3.

135
Properties of Normal
1 If the normal at the point P (at12, 2at1) meets the parabola at
2 y
Q (at22, 2at2), then t2 = –t1 – t
1
2
Let equation of parabola be y = 4ax.
P(at12,2at1)
Equation of normal at P is
y = – t1x+2at1+at13 x
Point Q lies on the normal, so
2at2 = –at1t22+2at1+at13
2a(t2–t1) = –at1(t22–t12) Q(at22,2at2)
2 = –t1(t2+t1) ( t1 ≠ t2)
2
∴ t2 = – t1 – t
1

2 If the normal at the points (at12, 2at1) and (at22, 2at2) meet on the parabola y2 = 4ax, then t1t2 =2.

Let the equation of normal at (at12,2at1) and (at22, 2at2) be


y = –t1x+2at1+at13
and y = – t2x+2at2+at23
meet the parabola y2 = 4ax at (at32,2at3) then
2 2
t3 = –t1– t and t 3 = –t2 – t
1 2

2 2
∴ – t1 – t = – t 2 – t
1 2

⎛1 1⎞ ⎛t −t ⎞
t2–t1 = 2 ⎜⎜ ⎟⎟ ⇒ t 2 − t1 = 2⎜⎜ 2 1 ⎟⎟
⎝ t1 t 2 ⎠ ⎝ t1t 2 ⎠
⇒ t1t2 = 2
3 No normal other than axis passes through focus.

Let equation of normal be y = mx–2am–am3


passes through (a,0) ie. focus
∴ 0 = ma–2am–am3
0 = – am–am3
0= –am((1+m2) ⇒ m = 0 i.e. axis
1+m2 = 0 which is not possible.
Ex1 Three normals to y2 = 4x pass through the point (15,12). One of the normals is
(A) x+y = 27 (b) x+4y = 63 (c) 3x–y = 33 (d) y+3x = 51
Let equation of normal be y = mx–2am–am3. a=1
∴ y = mx–2m–m3
passes through (15,12)
12 = 15m – 2m–m3
136
m3–13m +12 = 0
(m–1) (m–3) (m+4) = 0
m = 1, 3, –4.
m =1 ⇒ y = x–3
m =3 ⇒ y = 3x–33
m =–4 ⇒ y + 4x = 72

Ans (c)

Ex2 The minimum distance between the curves x2+y2–12x+31 = 0 and y2 = 4x is


(a) 5 (b) 2 5 (c) 6– 5 (d) None of these.
Centre (6,0) radius = 36 + 0 – 31 = 5
Minimum distance obtained along the common normal.
y2 = 4x
Differentiate w.r.t.x
dy
2y =4
dx
dy 2
=
dx y
y1
slope of normal at (x1,y1) is –
2

y1 – 0 y1
Also slope of CQ = x –6 = –
1 2
⇒ y1 = 0 or x1 = 4
Points are (0,0), (4,4), (4,–4)
OC = 6
QC = 2 5
RC = 2 5

⎧OC – R = 6 – 5 ⎫
⎪⎪ ⎪⎪
⎨ QC – R = 2 5 – 5 = 5 ⎬
Minimum distance ⎪ ⎪ is 5
⎩⎪ RC – R = 2 5 – 5 = 5 ⎪⎭
Ans (a)

137
Important Properties :
* If the tangent and normal at any point ‘P’ of the parabola intersect the axis at T and N then ST
= SN=SP where S in the focus.
y

P
Q
x
T S N

* The portion of a tangent to a parabola cut off between the directrix & the curve subtends a right
angle at the focus.
∠ PSQ = 90°
* Any tangent to a parabola and the perpendicular on it from the focus meet on the tangent at the
vertex.
∠ PQS = 90°
y

Q P

x
S

* If the tangents at A and B meet in P then PA and PB subtends equal angles at the focus S.
(SP)2 = SA×SB
Δ SAP~ Δ SPB
∠ PSA = ∠ PSB.

S
P

* The area of the triangle formed by three points on a parabola is twice the area of the triangle formed
by the tangents at these points.

138
PRACTICE QUESTIONS

1. The normal at the point (2, 4) of the parabola y2 = 8x meets the parabola at the point

a. (18, –12) b. (12, – 18) c. (12, 18) d. (18, 12)

2. If y+b = m1 (x+a) and y+b = m2 (x+a) are two tangents to the parabola y2 = 4ax, then

a. m1 m2 = 1 b. m1m2 = –1 c. m1+m2 = 0 d. m1 – m 2 = 0

3. If normals at the ends of the double ordinate x = 4 of parabola y2 = 4x meet the curve again
in P and P′ respectively, then P =

a. 10 b. 6 c. 12 d. 18

4. Radius of the largest circle which passes through the focus of the parabola y2 = 4x and
contained in it, is

a. 2 b. 4 c. 6 d. 8

5. If the normal at (1, 2) on the parabola y2 = 4x meets the parabola again at the point (t2, 2t),
then t is

a. 3 b. 1 c. 2 d. –3
P′
2a a
+ 3
6. If x = my+c is a normal to the parabola x2 m
= 4ay,
mthen c=

2a a
a. 2am+am3 b. c. –2am–am3 d. – – 3
m m

ANSWERS

1. a 2. b 3. c 4. b 5. d 6. c

139
140
PARABOLA - VII

Conormal points:
Let P (h,k) be a point and equation of parabola be y2 = 4ax.
Equation of normal is
y = mx–2am–am3 y
If passes through (h,k) so
k =mh –2am–am3
M
am3+2am–mh+k = 0
am3+m(2a–h) +k = 0 P(h, k)
Suppose m1, m2, m3 are the roots of this equation x
∴ m1+m2+m3 = 0
2a – h N
m1m2+m2m3+m3m1 = Q
a
k
m1 . m 2 . m 3 = –
a
So maximum three normal say PM, PN, PQ drawn through P. Points M, N, Q are called
co-normal points.
 The algebraic sum of ordinates of the conormal points is zero.
Let the coordinates of conormal points be M(am12,–2am1), N(am22,–2am2) and
Q (am32, –2am3). The ordinates of these points
y1+y2+y3 = – 2am1 –2am2 –2am3
= – 2a(m1+m2+m3)
=0
⇒ y1+y2+y3 = 0
 Centroid of the triangle formed by conormal points lies on the axis of parabola.
Let coordinates of conormal points be M(x2,y1), N(x2,y2) Q(x3,y3)
⎛ x1 + x 2 + x 3 y1 + y 2 + y 3 ⎞ ⎛ x 1 + x 2 + x 3 ⎞
Then centroid is ⎜ , ⎟=⎜ ,0 ⎟
⎝ 3 3 ⎠ ⎝ 3 ⎠
Since sum of ordinates is zero. Therefore centroid lies on the axis of parabola.
 Normal drawn from a point P(h,k) to the parabola are real and distinct if h>2a.
m12+m22+m32>0
⇒ (m1+m2+m3)2–2(m1m2+m2m3+m1m3) > 0
y
2( 2a – h )
⇒ 0– >0
a
⇒ 2a–h<0 S
x
⇒ h>2a (a,0) (2a,0)
This shows that position of point
(h,k) should be in shaded region.
 Equation of a circle passing through the conormal points
Let M(am12,–2am1), N(am22, –2am2) and Q (am32,–2am3)be three points on the parabola y2 =

141
4ax. These three normals passes through point p(h,k)
∴ am3+(2a–h)m+k = 0________________(1)
⇒ m1+m2+m3 = 0
2a – h
m1m2+m2m3+m1m3 =
a
k
m1 m 2 m 3 = –
a
Let equation of circle be x2+y2+2gx+2fy+c = 0
If the point (am2,–2am) lies on the circle then
(am2)2+(–2am) 2+2g(am2)+2f(–2am)+c = 0
a2m4+4a2m2+2gam2–4afm+c = 0
a2m4+(4a 2m2+2ga)m2–4afm+c = 0_______________ _(2)
This equation has four roots say m1, m 2, m3, m 4 such
that the circle passes through the points M(am12,–2am1)
N(am22,–2am2), Q(am32,–2am3)and S(am42,–2am4)
∴ m1+m 2+m 3+m 4 = 0 (From equation (2))
0 + m4 = 0 (Since m1+m2+m3 = 0)
∴ m4 = 0

⇒ S(0,0)
Therefore circle passes through origin.
∴ c= 0
Now equation (2) is
a2m4+(4a2+2ga)m2–4afm = 0 ( ÷ am) y
am3+(4a+2g)m –4f = 0 M
Now this equation is identical with equation (1)
O P(h, k)
x
a 2a – h k
= =
a 4a + 2g – 4f N
Q
k
⇒ 2g = – (2a+h), 2f = –
2
Equation of circle is
k
x2+y2 – (2a+h) x – y=0
2

142
PARABOLA - VIII
Chord of Contact
Let PA and PB be tangents drawn through the point P (h,k).
Equation of tangent at A is
yy1 = 2a (x+x1)
Equation of tangent at B is
yy2=2a(x+x2)
Both lines pass through (h,k)
∴ ky1 = 2a (h+x1)_____________(1)
ky2 = 2a (h+x2)_______________(2)
Hence A(x1,y1) and B(x2,y2) lie on
ky = 2a(x+h) or T = 0
ie. equation of chord AB.
Equation of chord whose midpoint (x1,y1) is given :
S ≡ y2 –4ax y
S1 ≡ y12–4ax1
T ≡ yy1 –2a(x+x1)
Equation of AB is T = S1
Reflection Property of Parabola Module – 7 (x1,y1)
x
Let PQ be tangent of the parabola y2 = ax at point P(at2,2at) O
Equation of tangent at P is ty = x+at2 y
Q is the point of intersection of tangent and x–axis
So Q (–at2,0)
A
∴ SQ = SA+AQ = a+at2 = a(1+t2) P(h, k) (x1,y1)
y
SP = PM = at2+a = a(1+t2) x
O
∴ SP = SQ (x2,y2) M P
I
∠ SPQ = ∠ SQP B
N
Also, ∠ MPQ = ∠ PQS (alternate angles) Q x
Hence ∠MPQ = ∠QPS A S(a, 0)

Also ∠IPN = ∠NPS


So PN normal bisects ∠IPS

Therefore PI is incident ray then PS is reflected ray. So any ray incident parallel to axis of the
parabola after reflection it passes through focus.
y
Example – 1 If three normals can be drawn to the parabola
y2–2y = 4x–9 from the point (a, b), then the range of
the value of a is
S
a. (2, ∞ ) b. (1, ∞ ) (3,1) (3,1) (4,1)
c. (– ∞ ,–2) d. (4, ∞ )
x

Solution: y2–2y = 4x–9

143
(y – 1)2 = 4(x – 2)
For real three normal’s as h > 2a
a>4
Ans: d

Example 2 A circle and a parabola y2 = 4ax intersect at four points. The algebraic sum of the
ordinates of the four points is
a. 0 b. 1 c. –1 d. 4

Solution: Let equation of circle be x2+y2+2gx+2fy+C = 0


and equation of parabola is x = at2, y = 2at
Solving it, we get a2t4+4a2t2+2agt2+4aft+c = 0
a2t4+2a(2a + g)t2+4aft+c = 0
t1+t2+t3+t4 = 0
Or 2at1+2at 2+2at3+2at 4 = 0
y1+y2+y3+y4 = 0
Ans: a

Example: 3 Maximum number of common normals of y2 = 4ax and x2 = 4by is


a. 3 b. 4 c. 6 d. 5

Solution: Normals of y2 = 4ax and x2 = 4by in slope form are


y = mx –2am –am2
b
y = mx + 2b +
m2
b
For common normal 2b + = –2am –am2
m2
am4+2am3+2bm2+b = 0
This mean there can be atmost 4 common normals
Ans: b
Example: 4 The curve described parametrically by x = t2+t+1, y = t2–t+1 represents
a. a pair of straight lines b. an ellipse
c. a parabola d. a hyperbola
Solution: x+y = 2(t2+1) and x–y = 2t
2
x+y x–y
∴ = +1
2 2
x2+y2–2xy+4 = 2x+2y
x2+y2–2xy–2x–2y+4 = 0
1– 1– 1
Δ = – 11– 1 =3–5–2 = – 4 ≠ 0
– 1 – 14

144
h2–ab = 1–1 = 0 ∴ It is a parabola
Ans: c

Example: 5 If x+y = k is normal to y2 =12x, then k is


a. 3 b. 9 c. –9 d. –3

Solution: Normal to y2 = 12x is y = mx–6m–3m3


m = –1
y = –x+6+3
x+y = 9
Ans: b

Example: 6 The equation of the common tangent touching the circle (x–3)2+y2 = 9 and the parabola
y2 = 4x above the x-axis is
a. 3 y = 3x + 1 b. 3y = – (x + 3)
c. 3y = x + 3 d. 3y = – (3x + 1)

1
Solution: Equation of tangent to the parabola is y = mx + and
m
equation of tangent to the circle is y = m(x–3) ± 3 1 + m 2
1
both the equation are identical i.e. = –3m ± 3 1 + m 2
m
2
1
m
+ 3m (
= 32 1 + m 2 )
1
+ 9m 2 + 6 = 9 + 9m 2
m2
1
1=3m2 m=±
3
Equation of common tangent is 3y = x + 3 (tangent lying above x–axis)
Ans: c

145
146
PARABOLA -IX
Exercise: 1 A tangent PT is drawn at the point P(16, 16) to the parabola y2 = 16x. PT tangent
intersect the x-axis at T. If S be the focus of the parabola, then ∠TPS is equal to
1 π 1 1 3
a. tan –1 b. c. tan –1 d. tan –1
2 4 2 2 4
Solution: ST = 4 + AT = 16+4 = 20
PS = 4+16 = 20
ΔTPS is isosceles triangle
16 – 0 4 2 tan θ θ
tan 2θ = = = θ 2θ
16 – 4 3 1 – tan 2 θ
S (4,0)
2 tan 2 θ + 3 tan θ – 2 = 0
(2 tan θ – 1)(tan θ + 2 ) = 0
1
tan θ = , tan θ = –2 (Not possible)
2
1
θ = tan –1 ( θ is acute angle)
2
Ans: a

Exercise: 2 If a, b, c are distinct positive real numbers such that the parabolas y2 = 4ax and y2
= 4c (x– b) will have a common normal, then
b b
a. 0< <1 b. <0
a –c a–c
b b
c. 1< <2 d. >2
a –c a –c

Solution: Equation of normals are


y = mx –2am – am3 ......(1)
3
y = m(x–b) –2cm – cm ......(2)
Equation 1 and 2 are identical then
–2am –am3 = –bm –2cm –cm3 ÷m
2a+am2 = b+2c+cm2
(a– c)m2 = b+2(c –a)
b
m2 = –2
a–c
b
m=± –2
a–c

147
b
For m be real >2
a –c
Ans: d

Exercise: 3 If AB be a chord of the parabola y2 = 4ax with vertex at A. BC is perpendicular to


AB such that it meets the axis at C. The projection of the BC on the axis of parabola is
a. 2a b. 4a c. 8a d. 16a

Solution: Let coordinates of B be (x, y) (x,y)


B
BD y
In ΔABD, tan θ = =
AD x
BD
In ΔBCD, tan (90 – θ ) =
o
o 90-O
A x D C
DC
y 4ax
∴ DC = y. = = 4a
x x
Ans: b

Exercise: 4 A circle is descirbed whose centre is the vertex and whose diameter is three-quarters
of the latus rectum of the parabola y2 = 4ax. If PQ is the common-chord of the circle and the
parabola and L1L2 is the latus rectum, then the area of the trapezium PL1L2Q is

2+ 2 2
a. a b. 4a 2 c. d.
2 2 2a 2 3 2a 2

Solution: Centre of circle (0, 0) D 3 2a 2

3
diameter = .4a = 3a B
4 P L1

2
9a
Eqn of circle x 2 + y 2 = .....(1)
4 O
Eqn of parabola y2 = 4ax .....(2)
coordinates of P and Q, we get after solving (1) and (2).
q L2
9a 2
x + 4ax =
2

4
2
5a 5a
(x+2a)2 = x = –2a ±
2 2
a – 9a
x= , ( notpossible)
2 2

148
a a
P , 2a , Q ,– 2a y = ± 2a
2 2
PQ = 2 2a , L1L2 = 4a
1
Area of trapezium = (PQ + L1L 2 ) x distance between them.
2
1
(
= 2 2 a + 4a x a –
2
)a
2

=
( 2+2 2
a
)
2
Ans: a

Exercise: 5 From the point (15, 12) three normals ae drawn to the parabola y2 = 4x, then centroid
of triangle formed by three co-normal points is
26
a. (5, 0) b. (5, 4) c. (9, 0) d. ,0
3
Solution: Let equation of normal be y = –tx+2t+t3
It passes through (15, 12). So 12 = –15t+2t+t3
t3 –13t –12 = 0
(t + 1) ( t+ 3) (t – 4) = 0
t = –1, –3, 4
2
Points are (at , 2at) i.e. (1, –2), (9, –6), (16, 8) y
1 + 9 + 16 – 2 – 6 + 8 26 (D) x-2=0
Centroid is , = ,0
3 3 3 P y=4
Ans: d
Exercise: 6 A ray of light travels along a line y = 4 and strikes
the surface of a curve y2 = 4(x+y) then equation of the (-1,2) (0,2)
line along reflected ray travel is
a. x+1 = 0 b. y–2 = 0
c. x=0 d. x–2 = 0
2 x
Solution: y –4y = 4x x' O
(y–2)2 = 4(x+1)
Focus (0,2) y'

Incident ray is parallel to axis of the parabola, so reflected ray passes through focus (0,2) i.e. x = 0

Exercise: 7 Let P be a point of the parabola y2 = 3(2x–3) and M is the foot of perpendicular drawn from
P on the directrix of the parabola, then length of each side of an equilateral triangle SMP, where S is
focus of the parabola is
a. 6 b. 8 c. 10 d. 11

149
Solution: Equation of parabola is
3
y2=6 x –
2
y M
focus S(3,0)
P
equation of directrix x = 0
3 3 2
P + t ,3t
2 2 x' x
A S
Coordinates of M (0,3t)
MS = 9 + 9t 2
3 3 2
MP = + t , But MS = MP y'
2 2
9 9 2 9 2
9+9t2 = + t + t
4 4 4
2
t =3
Length of side = 6
Ans: a
PRACTICE QUESTIONS
±
1. The point (2a, a) lies inside the region bounded by the parabola x2=4y and its latus rectum. Then
a. 0<a≤1 b. 0<a<1 c. 0≤a≤1 d. a<1
2
2. Two perpendicular tangents to y =4ax always intersect on the line
a. x+a=0 b. x–a=0 c. x+2a=0 d. y+2a=0
2 2 2
3. C1: y =8x and C2:x +y =2. Then
a. C1 and C2 have only two common tangents which are mutually perpendicular
b. C1 and C2 have two common tangents which are parallel to each other.
c. does not have any common tangent.
d. C1 and C2 have four common tangents.
4. Two common tangents to the circle x2+y2=2a2 and y2=8ax are
a. y= (x+a) b. x= ± (y+2a) c. y= ± (x+2a) d. x= ± (y+a)
5. The number of points with integral coordinates that lie in the interior of the circle x2+y2=16 and the
parabola y2=4x are
a. 6 b. 8 c. 10 d. 12
2 2
6. The vertex of the parabola x +y –2xy –4x+4=0 is at
⎛ 1 1⎞ ⎛ 1 1⎞
a. ⎜ – ,– ⎟ b. (–1, –1) c. (1, 1) d. ⎜+ , ⎟
⎝ 2 2⎠ ⎝ 2 2⎠
7. The length of the latus rectum of the parabola 2{(x–a)2 + (y–a)2}=(x+y)2 is
a. 2a b. 2a c. 2 2a d. 3 2a
2
8. The point on y =4ax nearest to the focus is

150
a
a. (0, 0) b. (a, 2a) c. (a, –2a) d. ( , a)
4
2
9. The angle between the tangents drawn from the origin to the parabola y =4a(x–a) is
1
a. 45° b. 60° c. 90° d. tan–1
2
10. The circle x2+y2+2λx=0, λεR, touches the parabola y2=4x externally. Then
a. λ>0 b. λ<0 c. λ>1 d. none of these

ANSWERS
1. b 2. a 3. a 4. c 5. b
6. d 7. c 8. a 9. c 10. a

151
152
PARABOLA - X
Example: 1 If the chord of contact of tangent from a point P to the parabola y2 = 4ax touches the parabola
x2 = 4by. The locus of P is
a. Parabola b. Hyperbola c. ellipse d. Circle

Solution: Let the point P be (h,k) then equation of chord of contact is ky = 2a(x+h).................(1)
Now this chord is tangent of parabola x2 = 4by
x2 = 4by
Equation of tangent xx1-2 by1 = 2by ------------------(2)
From (1) and (2) we get
4a b 4a b
x1= and y1=
k k

So, Equation of parabola becomes

4ab = –hk
Locus of (h,,k) is xy = –2ab. i.e. Hyperbola.
Ans: b . xy= 4ab
∴ΔPRQ
Example: 2 Let P and Q be points (4, –4) and (9, 6) on the parabola y2 = 4a(x–b). R is a point on the
parabola so that area is maximum, then
a. ∠PRQ = 900 b. the point R is (4,4)
1
c. the point R is ,1 d. None of these
4
Solution: (4,–4) lies on y2 = 4a(x–b) 16=4a(4–b)
(9,6) lies on y2 = 4a(x–b) 36=4a(9–b) Q (9.6)

16 4 b
a36==1 9 b b = 0
R

y2 = 4x
Let R be (t2,2t) on the Parabola thus
4 –4 1
1
area =2 9 { ( )
6 1 = 1 4(6 – 2 t ) + 4 9 – t 2 + 18t – 6 t 2 }
t2 2t 1 2

=
1
2
( )1
(
24 – 8t + 36 – 4t 2 + 18t – 6t 2 = – 10t 2 + 10t + 60
2
) (41-y)

153
2
1
+ 30 +
5 125
⇒ A = −5(t − 1 / 2) +
2
= –5(t –t–6) = –5 t –
2
2 4
4
1
Area is maximum when t =
2
1
Coordinates of R ,1
4
Ans: c

Example: 3 Minimum area of circle which touches the parabola’s y = x2+1 and y = x2-1 is
9π π 7π 9π
a. sq.unit b. sq.unit c. sq.unit d. sq.unit
32 4 32 16
Solution: y = x2+1 and y2 = x+1 are symmetrical about y = x
tangent at point. A and B are parallel to the line y = x
y = x2+1 y2 = x-1
dy 1
= 2x = 1 y=
dx 2
1 5 y = x2+1
x= x= m2 y=x
2 4 B
4
5 y2 = x-1
y= A
4
1 5 5 1
A , B ,
2 4 4 2
2 2
1 5 5 1 3 2
AB = – + – =
2 4 4 2 4
2
3 2 9π
Area of circle = πr 2 = π = sq.unit
8 32
Ans: a

Example: 4 The equation of the common tangents to the parabola y = x2 and y = –(x–2)2 is / are
a. y = 4(x–1) b. y=0 c. y = –4(x–1) d. y = –30x–50

Solution: Let y = mx+c is tangent to y = x2


∴ mx+c = x2 x2–mx–c = 0 has equal roots
m2+4c = 0

y = mx– is tangent to y = –(x–2)2 also

154
m2
mx– = –x2+4x–4
4
m2
x2+(m–4)x+4– = 0 has equal roots
4

∴ (m–4)2 –4

m2+16–8m–16+m2 = 0
m2–4m = 0
m = 0, 4
Equation of tangent are y = 0 and y = 4x–4
Ans: a,b
Example 5.(3,0) is the point from which three normals are drawn to the parabola y2 = 4x which meet the
parabola in the points P, Q and R then
Column I Column II
i. Area of ΔPQR a. 2 R

5
ii. Radius of circum circle of ΔPQR b.2
m 2 P
4– =0
4 5 Q
iii. Centroid of ΔPQR c. ,0
2

2
iv. Circum centre of ΔPQR d. ,0
3
Solution: Equation of normal is y = mx–2m–m3
It passes through (3,0), so 3m–2m–m3 = 0
m(1–m2) = 0
m = 0, 1, –1
2
Points are given by (m , –2m)
i.e. P(0,0), Q(1,–2), R(1,2)
0 0 1
1
area of ΔPQR = 2 1 – 2 1 = 2sq.units
1 2 1

abc 5 . 5 .4 5
R= = =
4Δ 4 .2 2
2
Centroid ΔPQR = ,0
3

155
5
Circum centre ,0
2
Comprehension based Questions (Exampels 6 to 8)
Comprehension 1
Consider the circle x2+y2 = 9 and the parabola y2 = 8x. They intersect at P and Q in the first and the
fourth quadrants, respectively. Tangents to the circle at P and Q intersect the x-axis at R and tangents
to the parabola at P and Q intersect the x-axis at S.

Example 6.The ratio of the area of the triangles PQS and PQR is
a. 1: 2 b. 1:2 c. 1:4 d. 1:8

Solution: Point of intersection of circle & parabola


x2+8x–9 = 0
(x+9) (x–1) = 0
X = 1, –9 (not possible)
Y = ± 22 2 5
(
P 1,2 2 , ) (
Q 1,–2 2 ) P

Tangent to the parabola at P is 2 2 y = 4(x+1)


S(–1,0)
Tangent to the circle at P is x+ 2 2 y = 9 S TR R

R(9,0)
Q
1
xPQxST ST 2 1
arΔPQS 2
= = = =
arΔPQR 1 xPQxRT RT 8 4
2
Ans: c

Example 7. The radius of the circum circle of the triangle PRS is


a. 5 b. 3 3 c. 3 2 d. 2 3
1 1
Solution: area ΔPRS = Δ = xSRxPT = x10x 2 2 = 10 2
2 2
abc 10.6 2 .2 3
R= = =3 3
4Δ 4.10 2
Ans: b
Example 8.The radius of the in circle of the triangle PQR is
8
a. 4 b. 3 c. d. 2
3

156
1 1
= PQxRT . 4 2 .8
Δ 2 2 16 2
= = = =2
Solution: r = S PR + RQ + QP 6 2 + 6 2 + 4 2 8 2
2 2
Ans: d

COMPREHENSION 2 (EXAMPLES 9 TO 11)


If y = x is tangent to the parabola y = ax2+c
9. If a = 2, then the value of c is
1 1 1
a. b. c. d. 1
2 4 8
10 If (1,1) is point of contact then ‘a’ is
1 1 1
a. 1 b. c. d.
2 3 4
11 If c = 2 then point of contact is
1 1
a. (4,4) b. (2,2) c. (8,8) d. ,
2 2
9. Solution: y = ax2+c

dy 1 Point of contact of the tangent is ⎛ 1 , 1 + c⎞⎟ since it lies on y = x


= 2ax = 1 ⇒ x = ⎜
dx 2a ⎝ 2a 4a ⎠
1
∴ thus c = for a = 2
8
Ans: c
1
10. If (1,1) is point of contact then a =
2
Ans: b

11. If c = 2, then point of contact is ⎛⎜ 1 , 1 + 2⎞⎟


⎝ 2a 4a ⎠
Since it lies on the line y = x,
1 1 1
= +2⇒a =
2a 4a 8
∴ point of contact is (4,4)
PRACTICE QUESTIONS
1. The point P on the parabola y =4ax for which ⏐PR–PQ⏐ is maximum, where R (–a, 0), Q (0, a) is
2

a. (4a, –4a) b. (4a, 4a) c. (a, 2a) d. (a, –2a)

157
2. The shortest distance between the parabola y2=4x and the circle x2+y2+6x–12y+20=0 is
a. 4 2 –5 b. 4 2 +5 c. 3 2 +5 d. 3 2 –5
2
3. If normals are drawn from a point p (h, k) to the parabola y =4ax, then the sum of the intercepts
which the normals act off from the axis of the parabola is
a. 4(h+0) b. 3(h+c) c. 2(h+a) d. (h+a)
4. If a ≠ 0 and the line 2px+3qy + 4r =0 passes through the points of intersection of the parabolas
y2=4ax and x2=4ay, then
a. r2+(3p+2q)2=0 b. r2+(2p+3q)2=0 c. r2+(3p–2q)2=0 d. r2+(2p–2q)2=0
5. The equation of the tangent at the vertex of the parabola x2+4x+2y=0 is
a. x=2 b. x=–2 c. y=–2 d. y=2
2 2 2
6. The common tangent to the parabolas y =4ax and x =4ax and x =32ay is
a. x+2y–4a=0 b. x+2y+4a=0 c. x–2y+4a=0 d. x–2y–4a=0
7. The shortest distnae between the parabolas y2=4x and y2=2x–6 is
a. 5 b. 2 c. 3 d. none of these
8. The largest value of a for which the circle x +y =a falls totally in the interior of the parabola y2=4(x+4)
2 2 2

is
a. 4 b. 4 3 c. 3 3 d. 2 3
Multiple choice questions with one or more than one correct answer.
9. Let P(x1, y1) and Q(x2, y2), y1<0, y2 ,0, be the end points of the latus rectum of the ellipse x2+4y=4.
The equations of parabolas with latus rectum PQ are
a. x2+2 3 y=3+ 3 b. x2–2 3 y=3+ 3
c. x2+2 3 y=3– 3 d. x2–2 3 y=3– 3
10. The tangent PT and the normal PN to the parabola y2=4ax at a point P on it meet its axis at point T
and N, respectively. The locus of the centroid of the triangle PTN is a parabola whose
⎛ 2a ⎞
a. vertex is ⎜ ,0 ⎟ b. directrix is x=0
⎝ 3 ⎠
2a
c. latus rectum is d. focus is (a, 0)
3
Match the following :
11. Consider the parabola y2=12x
Column I Column II
A Equation of tangent can be p. 2x+y–6=0
B. Equation of normal can be q. x–2y–12=0
C. Equation of chord of contact w.r.t. any point on the directrix r. 2x–y=36
D. Equation of chord which subtends right angle at the vertex s. 3x–y+1=0
Assertion and Reasoning
x2
12. Statement 1 : The curve y=– +x+1 is symmetric with respect to the line x =1.
2
Statement 2 : A parabola is symmetric about its axis.
(A) Statement 1 is True, Statement 2 is True; Statement 2 is a correct explanations

158
for statement 1.
(B) Statement 1 is True, statement 2 is true, statement 2 is not a correct explanation
for statement 1.
(C) Statement 1 is true, statement 2 is false.
(D) Statement 1 is false, statement 2 is true.

ANSWERS
1. c 2. a 3 c 4. b 5 d
6. b 7. a 8. d 9. b, c 10. a, d
11. A → s, B → r, C → p, D → q 12. A

159
160

Das könnte Ihnen auch gefallen